Корень в степени как решать: Свойства корней (ЕГЭ 2022) | YouClever

{n}=\underbrace{b*b*b*…*b}_{n \; раз}=a. $$

Число \(n\) при этом называют показателем корня.

Если \(n=2\), то перед вами корень 2-й степени или обычный квадратный корень.

Если \(n=3\), то корень 3-й степени и т.д.

Операция извлечения корня n-й степени является обратной к операции возведения в n-ю степень.


Пример 1 $$ \sqrt[3]{27}=3 $$

Кубический корень из числа 27 равняется 3. Действительно, если число 3 возвести в 3-ю степень, то мы получим 27.


Пример 2 $$ \sqrt[4]{16}=2 $$

Корень 4-й степени из 16-и равен 2. Двойка в 4-й степени равна 16.


Пример 3 $$ \sqrt[3]{0}=0 $$

Если извлечь корень n-й степени из 0, всегда будет 0.


Пример 4 $$ \sqrt[3]{19}= ? $$

Мы не можем в уме подобрать такое число, которое при возведении в 3-ю степень даст 19. Если посчитать на калькуляторе, то получим \(2,668…\) – иррациональное число с бесконечным количеством знаков после запятой.

Обычно, в математике, когда у вас получается иррациональное число, корень не считают и оставляют так как есть \(\sqrt[3]{19}\).

Что же делать, если под рукой нет калькулятора, а нужно оценить, чему равен такой корень. В этом случае нужно подобрать справа и слева такие ближайшие числа, корень из которых посчитать можно:

$$ \sqrt[3]{8} \le \sqrt[3]{19} \le \sqrt[3]{27} $$ $$ 2 \le \sqrt[3]{19} \le 3 $$

Получается, что наш корень лежит между числами 2 и 3.


Содержание

Корень четной и нечетной степени

Надо четко различать правила работы четными и нечетными степенями. Дело в том, что корень четной степени можно взять только из положительного числа. Из отрицательных чисел корень четной степени не существует.

Корень нечетной степени можно посчитать из любых действительных чисел. Иногда в школьной программе встречаются задания, в которых требуется определить имеет ли смысл выражение:


Пример 5 $$ \sqrt[3]{-27}=-3 $$

Данное выражение имеет смысл, так как корень нечетной степени можно посчитать из любого числа, даже отрицательного.


Пример 6 $$ \sqrt[4]{-27} $$

Так как корень четной степени, а под корнем стоит отрицательное число, то выражение не имеет смысла.k} $$

Что такое квадратный корень? Формулы и Примеры

Что такое квадратный корень

Определение арифметического квадратного корня ясности не добавляет, но заучить его стоит:

Арифметическим квадратным корнем из неотрицательного числа a называется такое неотрицательное число, квадрат которого равен a.

Определение квадратного корня также можно представить в виде формул:
√a = x
x2 = a
x ≥ 0
a ≥ 0

Из определения следует, что a не может быть отрицательным числом. То есть то, что стоит под корнем — обязательно положительное число.

Чтобы разобраться, почему именно так и никак иначе, давайте рассмотрим пример.

Попробуем найти корень из √-16

Здесь логично предположить, что 4, но давайте проверим: 4*4 = 16 — не сходится.

Если — 4, то -4 * -4 = 16, (минус на минус всегда дает плюс).

Получается, что ни одно число не может дать отрицательный результат при возведении его в квадрат.

Числа, стоящие под знаком корня, должны быть положительными.

Исходя из определения, значение корня также не должно быть отрицательным

Здесь могут возникнуть резонные вопросы, почему, например, в примере x2 = 16, x = 4 и x = -4.

Разница между квадратным корнем и арифметическим квадратным уравнением

Прежде всего, чтобы разграничить эти два понятия, запомните:

  • x2 = 16 не равно  x = √16.

Это два нетождественных друг другу выражения.

  • x2 = 16 — это квадратное уравнение.
  • x = √ 16 — арифметический квадратный корень.

Из выражения x2 = 16 следует, что:

  • |x| = √16, это значит, что x = ±√16 = ±4, x1 = 4, x2 = -4.

Если две вертикальные палочки возле x вводят вас в замешательство, почитайте нашу статью о модуле числа.

В то же самое время, из выражения x = √16 следует, что x = 4.

Если ситуация все еще кажется запутанной и нелогичной, просто запомните, что отрицательное число может быть решением только в квадратном уравнении. Если в решении «минус» — есть два варианта:

 
  1. Пример решен неверно

  2. Это квадратное уравнение.

Если вы извлекаете квадратный корень из числа, то можете быть уверены, вас ждет «положительный» результат.

Давайте рассмотрим пример, чтобы окончательно выяснить разницу между квадратным корнем и квадратным уравнением.

Даны два выражения: 

 
  1. x2 = 36

  2. x = √36

Первое выражение — квадратное уравнение. 

|x| = √36
x1 = +6
x2 = -6.

Второе выражение — арифметический квадратный корень. 

√36 = 6
x = 6.

Мы видим, что результатом решения первого выражения стали два числа — отрицательное и положительное. А во втором случае — только положительное.

Запись иррациональных чисел с помощью квадратного корня

Иррациональное число — это число, которое нельзя представить в виде обыкновенной дроби.

Чаще всего, иррациональные числа можно встретить в виде корней, логарифмов, степеней и т.д.

Примеры иррациональных чисел:

√2 = 1,414213…;

π = 3,141592…;

e = 2,718281…. .

Чтобы упростить запись иррациональных чисел, математики ввели понятие квадратного корня. Давайте разберем пару примеров, чтобы увидеть квадратный корень в деле.

Дано уравнение: x2 = 2.

Сразу сталкиваемся с проблемой, поскольку очевидно, что ни одно целое число не подходит. 

Переберем числа, чтобы удостовериться в этом:

1 * 1 = 1,
2 * 2 = 4,
3 * 3 = 9.

Отрицательные числа дают такой же результат. Значит результатом решения не могут быть целые числа.

Решение следующее:
Строим график функции

y = x2.
Отмечаем решения на графике: -√2; √2.


Если попробовать извлечь квадратный корень из 2 с помощью калькулятора, то результат будет следующий: √2 = 1,414213… .

В таком виде ответ не записывают — нужно оставить квадратный корень.
x2 = 2.
x = √2
x = -√2. 

Извлечение корней

Решать примеры с квадратными корнями намного легче, если запомнить как можно больше квадратов чисел. Для этого воспользуйтесь таблицей — сохраните ее себе и используйте для решения задачек.

Таблица квадратов


Вот несколько примеров извлечения корней, чтобы научиться пользоваться таблицей:

  • 1. Извлеките квадратный корень: √289

Ищем в таблице число 289, двигаемся от него влево и вверх, чтобы определить цифры, образующие нужное нам число.

Влево — 1, вверх — 7.

Ответ: √289 = 17.

  • 2. Извлеките квадратный корень: √3025

Ищем в таблице число 3025.
Влево — 5, вверх —  5.

Ответ: √3025 = 55.

  • 3. Извлеките квадратный корень: √7396

Ищем в таблице число 7396.

Влево — 8, вверх — 6.

Ответ: √7396 = 86.

  • 4. Извлеките корень: √9025

Ищем в таблице число 9025.

Влево — 9, вверх — 5.

Ответ: √9025 = 95.

  • 5. Извлеките корень √1600

Ищем в таблице число 1600.

Влево — 4, вверх — 0.

Ответ: √1600 = 40.

Извлечением корня называется нахождение его значение.

Свойства арифметического квадратного корня

У арифметического квадратного корня есть 3 свойства — их нужно запомнить, чтобы проще решать примеры.

  • Корень произведения равен произведению корней
  • Извлечь корень из дроби — это извлечь корень из числителя и из знаменателя
  • Чтобы возвести корень в степень, нужно возвести в степень значение под корнем

Давайте потренируемся и порешаем примеры на все три свойства. Не забывайте обращаться к таблице квадратов. Попробуйте решить примеры самостоятельно, а для проверки обращайтесь к ответам.

Умножение арифметических корней

Для умножения арифметических корней используйте формулу:

Примеры:

 

Внимательно посмотрите на второе выражение и запомните, как записываются такие примеры.

Если нет возможности извлечь корни из чисел, то поступаем так:

 

  1. Если множителей больше двух, то решается примерно точно так, как и с двумя множителями:

Добрая напоминалочка

Чтобы решать примеры быстрее, не забывайте пользоваться таблицей квадратов.

 


Деление арифметических корней

Для деления арифметических корней используйте формулу:

Примеры:

 
  1. Ответ: смешанную дробь превращаем в неправильную (16 * 3) + 1 = 49





Выполняя деление, не забывайте сокращать множители. При делении арифметических корней, используйте правила преобразования обыкновенных дробей.

Возведение арифметических корней в степень

Для возведения арифметического корня в степень используйте формулу:

Примеры:



Эти две формулы нужно запомнить:


Повторите свойства степеней, чтобы без труда решать такие примеры.

Внесение множителя под знак корня

Вы уже умеете по-всякому крутить и вертеть квадратными корнями: умножать, делить, возводить в степень. Богатый арсенал, не правда ли? Осталось овладеть еще парой приемов и можно без страха браться за любую задачку.

А теперь давайте разберемся, как вносить множитель под знак корня.

Дано выражение: 7√9

Число семь умножено на квадратный корень из числа девять. 

Извлечем квадратный корень и умножим его на 7.

√9= 3.

7√9 = 7*3 = 21

В данном выражение число 7 — множитель. Давайте внесем его под знак корня. 

Запомните, что вносить множитель под знак корня обязательно нужно так, чтобы значение исходного выражения осталось неизменным. Иными словами, после наших манипуляций с корнем, значение выражения должно по-прежнему оставаться 21.

Вы помните, что (√a)2 = a

Тогда число 7 должно быть возведено во вторую степень. В этом случае значение выражения останется тем же. 

7√9 = √72* 9 = √49 * 9 = √49 * √9 = 7 * 3 = 21.

Формула внесения множителя под знак корня:

Запоминаем:

Нельзя вносить отрицательные числа под знак корня.

Потренируемся вносить множители. Попробуйте решить примеры самостоятельно, сверяясь с ответами.

 


Вынесение множителя из-под знака корня 

С тем, как вносить множитель под корень мы, кажется, разобрались. Но алгебра — такая алгебра, поэтому теперь неплохо бы и вынести множитель из-под знака корня.

Дано выражение в виде квадратного корня из произведения.

Вы уже наверняка без труда извлекаете квадратный корень из чего угодно, поэтому знаете, что делать.

Извлекаем корень из всех имеющихся множителей. 


В данном выражении квадратный корень мы можем извлечь только из 4, поэтому:


Таким образом множитель выносится из-под знака корня.

Давайте разберем примеры. Попробуйте вынести множители из-под знака корня самостоятельно, сверяясь с ответами.

 
  1. √28

    Раскладываем подкоренное выражение на множители 28 = 7*4.

    Извлекаем корень из 4. Множитель 7 оставляем под знаком корня.



  2. Ответ: по правилу извлечения квадратного корня из произведения,

    Так как вынесенный множитель должен стоять перед подкоренным знаком, то меняем их местами.

  3. Вынесите множитель из-под знака корня в выражении: √24

    Ответ: Раскладываем выражение под корнем на множители 24 = 6 * 4.


  4. Упростите выражение:

    Вынесем в двух последних выражения множитель из-под знака корня.

    Умножаем (-4 * 4) = -16. Все остальное выражение записываем в неизменном виде.

    Мы видим, что во всем выражении есть один общий множитель — √5.
    Выносим общий множитель за скобки:

    Далее вычисляем все, что в скобках:

 

Сравнение квадратных корней

Мы почти досконально разобрали арифметический квадратный корень, научились умножать, делить и возводить его в степень. Теперь вы без труда можете вносить множители под знак корня и выносить их оттуда. Осталось научиться сравнивать корни и стать непобедимым теоретиком.

Итак, чтобы понять, как сравнить два квадратных корня, нужно запомнить пару правил.

Если:

  • √a < √b, то a < b
  • √a = √b, то a = b

Давайте разберем на примере.

Сравните два выражения: √70 и 8√2

Первым делом преобразуем второе выражение: 8√2 = √64 * √2 = √64*2 = √128.

70 < 128.

Это значит, что √70  <  8√2.

Запоминаем

Чем больше число под знаком корня, тем больше сам корень.

Потренируйтесь в сравнении корней. Сверяете свои результаты с ответами.

 
  1. Сравните два выражения: √50 и 9√5

    Ответ: преобразовываем выражение 9√5.

    9√5 = √81 * √5 = √81*5 = √405

    50 < 405

    Это значит, что √50 < 9√5.


  2. Сравните два выражения: 6√5 и √18

    Ответ: преобразовываем выражение 6√5.

    6√5 = √36 * √5 = √36*5= √180

    180 > 18

    Это значит, что 6√5 > √18.


  3. Сравните два выражения: 7√12 и √20

    Ответ: преобразовываем выражение 7√12.

    7√12 = √49 * √12 = √49*12 = √588

    588 >20

    Это значит, что 7√12 > √20.

Как видите, ничего сложного в сравнении арифметических квадратных корней нет. 

Самое главное — выучить формулы и сверяться с таблицей квадратов, если значения корня слишком большие для легкого вычисления в уме.

Не бойтесь пользоваться вспомогательными материалами. Математика просто создана для того, чтобы окружить себя подсказками и намеками.

Когда вы почувствуете, что уже достаточно натренировались в решении примеров с квадратными корнями, можете позволить себе время от времени прибегать к помощи онлайн-калькуляторов. Они помогут решать примеры быстрее и быть эффективнее. 

Таких калькуляторов в интернете много, вот один из них.

Извлечение квадратного корня из большого числа

Вы уже наверняка познакомились и подружились с таблицей квадратов. Она — ваша правая рука. С ее помощью вы реактивно решаете примеры и, возможно, даже подумываете запомнить ее наизусть.

Но, как вы можете заметить, таблица заканчивается на числе 9801. А это, согласитесь, не самое крупное число из тех, что могут вам попасться в примере.


Чтобы извлечь корень из большого числа, которое отсутствует в таблице квадратов, нужно:

 
  1. Определить «сотни», между которыми оно стоит.

  2. Определить «десятки», между которыми оно стоит.

  3. Определить последнюю цифру в этом числе.

Извлечь корень из большого числа можно разными способами — вот один из них.

Извлечем корень из √2116.

Наша задача в том, чтобы определить между какими десятками стоит число 2116.

102 = 100

202 = 400

302 = 900

402 = 1600

502 = 2500 

Мы видим что, 2116 больше 1600, но меньше 2500.

Это значит, что число 2116 находится между 402и 502.

41, 42, 43, 44, 45, 46, 47, 48, 49.

Запомните лайфхак по вычислению всего на свете, что нужно возвести в квадрат.

Не секрет, что на последнем месте в любом числе может стоять только одна цифра от 1 до 0.


Как пользоваться таблицей

12 = 1

22 = 4

32 = 9

42 = 16 ⇒ 6

52 = 25 ⇒ 5

62 = 36 ⇒ 6

72 = 49 ⇒ 9

82 = 64 ⇒ 4

92 = 81 ⇒ 1

Мы знаем, что число 41, возведенное в квадрат, даст число, на конце которого — цифра 1.

Число, 42, возведенное в квадрат, даст число, на конце которого — цифра 4.

Число 43, возведенное в квадрат, даст число, на конце которого — 9.

Такая закономерность позволяет нам без записи «перебрать» все возможные варианты, исключая те, которые не дают нужную нам цифру 6 на конце.

Таким образом, у нас остаются два варианта: 442 и 462.

Далее вычисляем: 44 * 44 = 1936.

46 * 46 = 2116.

Ответ: √2116 = 46

Если такой способ показался не до конца понятным — можно потратить чуть больше времени и разложить число на множители. Если решить все правильно, получим такой же результат. 

Еще пример. Извлечем корень из числа √11664

Разложим число 11664 на множители: 

11666 : 4 = 2916

2916 : 4 = 729

729 : 3 = 243

243 : 3 = 81

11664

4

2916

4

729

3

243

3

81

81

Запишем выражение в следующем виде:


Извлечь квадратный корень из большого числа гораздо проще с помощью калькулятора. Но знать парочку таких способов «на экстренный случай» точно не повредит. Например, для контрольной или ЕГЭ.

Чтобы закрепить все теоретические знания, давайте ещё немного поупражняемся в решении примеров на арифметические квадратные корни.
 

Mathway | Популярные задачи

1 Оценить с использованием заданного значения квадратный корень 50
2 Оценить с использованием заданного значения квадратный корень 45
3 Вычислить 5+5
4 Вычислить 7*7
5 Разложить на простые множители 24
6 Преобразовать в смешанную дробь 52/6
7 Преобразовать в смешанную дробь 93/8
8 Преобразовать в смешанную дробь 34/5
9 График y=x+1
10 Оценить с использованием заданного значения квадратный корень 128
11 Найти площадь поверхности сфера (3)
12 Вычислить 54-6÷2+6
13 График y=-2x
14 Вычислить 8*8
15 Преобразовать в десятичную форму 5/9
16 Оценить с использованием заданного значения квадратный корень 180
17 График y=2
18 Преобразовать в смешанную дробь 7/8
19 Вычислить 9*9
20 Risolvere per C C=5/9*(F-32)
21 Упростить 1/3+1 1/12
22 График y=x+4
23 График y=-3
24 График x+y=3
25 График x=5
26 Вычислить 6*6
27 Вычислить 2*2
28 Вычислить 4*4
29 Вычислить 1/2+(2/3)÷(3/4)-(4/5*5/6)
30 Вычислить 1/3+13/12
31 Вычислить 5*5
32 Risolvere per d 2d=5v(o)-vr
33 Преобразовать в смешанную дробь 3/7
34 График y=-2
35 Определить наклон y=6
36 Перевести в процентное соотношение 9
37 График y=2x+2
38 График y=2x-4
39 График x=-3
40 Решить, используя свойство квадратного корня x^2+5x+6=0
41 Преобразовать в смешанную дробь 1/6
42 Преобразовать в десятичную форму 9%
43 Risolvere per n 12n-24=14n+28
44 Вычислить 16*4
45 Упростить кубический корень 125
46 Преобразовать в упрощенную дробь 43%
47 График x=1
48 График y=6
49 График y=-7
50 График y=4x+2
51 Определить наклон y=7
52 График y=3x+4
53 График y=x+5
54 График 3x+2y=6
55 Решить, используя свойство квадратного корня x^2-5x+6=0
56 Решить, используя свойство квадратного корня x^2-6x+5=0
57 Решить, используя свойство квадратного корня x^2-9=0
58 Оценить с использованием заданного значения квадратный корень 192
59 Оценить с использованием заданного значения квадратный корень 25/36
60 Разложить на простые множители 14
61 Преобразовать в смешанную дробь 7/10
62 Risolvere per a (-5a)/2=75
63 Упростить x
64 Вычислить 6*4
65 Вычислить 6+6
66 Вычислить -3-5
67 Вычислить -2-2
68 Упростить квадратный корень 1
69 Упростить квадратный корень 4
70 Найти обратную величину 1/3
71 Преобразовать в смешанную дробь 11/20
72 Преобразовать в смешанную дробь 7/9
73 Найти НОК 11 , 13 , 5 , 15 , 14 , , , ,
74 Решить, используя свойство квадратного корня x^2-3x-10=0
75 Решить, используя свойство квадратного корня x^2+2x-8=0
76 График 3x+4y=12
77 График 3x-2y=6
78 График y=-x-2
79 График y=3x+7
80 Определить, является ли полиномом 2x+2
81 График y=2x-6
82 График y=2x-7
83 График y=2x-2
84 График y=-2x+1
85 График y=-3x+4
86 График y=-3x+2
87 График y=x-4
88 Вычислить (4/3)÷(7/2)
89 График 2x-3y=6
90 График x+2y=4
91 График x=7
92 График x-y=5
93 Решить, используя свойство квадратного корня x^2+3x-10=0
94 Решить, используя свойство квадратного корня x^2-2x-3=0
95 Найти площадь поверхности конус (12)(9)
96 Преобразовать в смешанную дробь 3/10
97 Преобразовать в смешанную дробь 7/20
98 Преобразовать в смешанную дробь 2/8
99 Risolvere per w V=lwh
100 Упростить 6/(5m)+3/(7m^2)

Действия со степенями и корнями

1. При умножении степеней с одинаковыми основаниями показатели складываются, а основание остаётся прежним:

.

Например, .

2. При делении степеней с одинаковыми основаниями показатели степеней вычитаются, а основание остаётся прежним:

.

Например, .

3. При возведении степени в степень показатели степеней перемножаются, а основание остаётся прежним:

.

Например, .

4. Степень произведения равна произведению степеней множителей:

.

Например, .

5. Степень частного равна частному степеней делимого и делителя:

.

Например, .

Пример 1. Найти значение выражения

.

Решение. В данном случае в явной форме ни одно из свойств степени с натуральным показателем применить нельзя, так как все степени имеют разные основания. Запишем некоторые степени в другом виде:

(степень произведения равна произведению степеней множителей),

(при умножении степеней с одинаковыми основаниями показатели складываются, а основание остаётся прежним, при возведении степени в степень показатели степеней перемножаются, а основание остаётся прежним).

Теперь получим:

В данном примере были использованы первые четыре свойства степени с натуральным показателем.

Свойства степеней и корней интенсивно используются при упрощении выражений в задачах математического анализа, например, для нахождения производной параметрически заданной функции и производной функции, заданной неявно.

Имеют место следующие тождества:

1) ;

2) ;

3) .

Выполнить действия со степенями самостоятельно, а затем посмотреть решения

Пример 2. Найти значение выражения

.

Пример 3. Найти значение выражения

.

1. Корень k-й степени из произведения неотрицательных чисел равен произведению корней той же степени из сомножителей: , где (правило извлечения корня из произведения).

2. Если , то (правило извлечения корня из дроби).

3. Если , то (правило извлечения корня из корня).

4. Если , то (правило возведения корня в степень).

5. Если , то , где , т. е. показатель корня и показатель подкоренного выражения можно умножить на одно и то же число.

6. Если , то , т. е. большему положительному подкоренному выражению соответствует и большее значение корня.

7. Все указанные выше формулы часто применяются в обратном порядке (т. е. справа налево). Например:

(правило умножения корней),

(правило деления корней),

.

8. Правило вынесения множителя из-под знака корня. При .

9. Обратная задача — внесение множителя под знак корня. Например,

10. Уничтожение иррациональности в знаменателе дроби. Рассмотрим некоторые типичные случаи.

а) , так как .

Например, .

б)

Например,

в)

и т. д.

Другие темы в блоке «Школьная математика»

Как решать примеры с корнями

Корнем n степени из числа называют такое число, которое при возведении в эту степень даст то число, из которого извлекается корень. Чаще всего, действия производятся с корнями квадратными, которые соответствуют 2 степени. При извлечении корня часто невозможно найти его явно, а результатом является число, которое невозможно представить в виде натуральной дроби (трансцендентное). Но используя некоторые приемы, можно значительно упростить решение примеров с корнями.Вам понадобится

Если не требуется абсолютная точность, при решении примеров с корнями воспользуйтесь калькулятором. Чтобы извлечь из числа квадратный корень, наберите его на клавиатуре, и просто нажмите соответствующую кнопку, на которой изображен знак корня. Как правило, на калькуляторах берется корень квадратный. Но для вычисления корней высших степеней, воспользуйтесь функцией возведения числа в степень (на инженерном калькуляторе).4=(-2)∙ (-2)∙ (-2)∙ (-2)=16. Для извлечения квадратного корня нацело, когда это возможно, воспользуйтесь таблицей квадратов натуральных чисел.

Если же рядом нет калькулятора, или требуется абсолютная точность в расчетах, используйте свойства корней, а также различные формулы для упрощения выражений. Из многих чисел можно извлечь корень частично. Для этого воспользуйтесь свойством, что корень из произведения двух чисел равен произведению корней из этих чисел √m∙n=√m∙√n.

Пример. Вычислите значение выражения (√80-√45)/ √5. Прямое вычисление ничего не даст, поскольку нацело не извлекается ни один корень. Преобразуйте выражение (√16∙5-√9∙5)/ √5=(√16∙√5-√9∙√5)/ √5=√5∙(√16-√9)/ √5. Произведите сокращение числителя и знаменателя на √5, получите (√16-√9)=4-3=1.

Если подкоренное выражение или сам корень возведены в степень, то при извлечении корня воспользуйтесь тем свойством, что показатель степени подкоренного выражения можно поделить на степень корня. Если деление производится нацело, число вносится из-под корня.4=5²=25.

Пример. Вычислить значение выражения (√3+√5)∙(√3-√5). Примените формулу разности квадратов и получите (√3)²-(√5)²=3-5=-2.

Корень n-ой степени

Вопросы занятия:

·  повторить, как извлекается корень n-ой степени из числа;

·  повторить свойства арифметического корня n-ой степени;

·  показать, как можно применить свойства корня при решении задач.

Материал урока

Определение.

Корнем n-ой степени из числа а называется такое число, энная степень которого равна А.

Говоря о корне энной степени нужно понимать, что показатель корня n является натуральным числом.

Вам хорошо известен такой частный случай корня n-ой  степени, как корень второй степени, то есть квадратный корень из числа. Показатель корня в этом случае не пишут.

Определение.

Квадратным корнем из числа называют такое число, квадрат которого равен числу а.

Например,

Ещё одним частным случаем является корень третьей степени, мы привыкли называть его корнем кубическим.

Например,

Вы могли задаться вопросом, почему.

Чтобы ответить на этот вопрос, рассмотрим два случая корня энной степени: где показатель корня является нечётным числом и где показатель корня является чётным числом.

Но чтобы избавиться от неоднозначности в вычислениях, договорились неотрицательный корень n-ой степени обозначать как «корень n-ой  степени из а». А отрицательный как «минус корень n-ой степени из а».

Исходя из этой договорённости и становится понятно, что

Из рассмотренных случаев можем сделать заключение, что:

Пример.

Вернёмся к определению корня энной степени

В первую запись вместо можно подставить:

Тогда получаем свойство, которым очень удобно пользоваться при вычислении корня энной степени.

Но ведь корень чётной степени мы договорились считать числом неотрицательным. Поэтому чтобы не возникало путаницы при вычислении корней, вместо этого свойства мы пользовались двумя:

Пример.

Вы видите, что корень энной степени из любого неотрицательного числа а имеет смысл при любых Эн и принимает неотрицательные значения. Такой корень называют арифметическим корнем n-ой степени из числа а.

Любой корень можно выразить через арифметический.

Корни чётных степеней всегда являются арифметическими, ведь подкоренное выражение у них является числом неотрицательным, и под их значениями мы условились понимать неотрицательные числа.

Это же касается корней нечётных степеней из неотрицательных чисел.

А вот, например, корни нечётных степеней из отрицательных чисел можно записать так, при этом вынеся минус перед корнем.

Таким образом, при работе с любым корнем энной степени можно всегда перейти к рассмотрению арифметического корня.

Также при работе с корнями энной степени очень важно знать ещё несколько свойств. Напомним их.

Все эти свойства пригодятся вам при работе с выражениями, содержащими знак корня.

Итоги урока

На этом уроке мы с вами вспомнили, как извлекать корень n-ой степени из числа. Напомнили, что корень n-ой степени из неотрицательного числа считают числом неотрицательным, и называют «арифметическим корнем n-ой степени».

Так корень чётной степени из неотрицательного числа всегда является корнем арифметическим и поэтому равен числу неотрицательному. А корня чётной степени из отрицательного числа не существует. Корень нечётной степени из неотрицательного числа равен числу неотрицательному. Например, корень третьей степени из двадцати семи равен трём.

Ну, а корень нечётной степени из отрицательного числа равен числу отрицательному. Его нужно выражать с помощью арифметического корня n-ой степени, при этом вынося минус из-под знака корня. Также мы напомнили свойства арифметического корня энной степени и применили их на конкретных примерах.

Возведение в степень и извлечение корня в Excel

Для извлечения корня в Excel и возведения числа в степень используются встроенные функции и математические операторы. Рассмотрим на примерах.

Примеры функции КОРЕНЬ в Excel

Встроенная функция КОРЕНЬ возвращает положительное значение квадратного корня. В меню «Функции» она находится в категории «Математические».

Синтаксис функции: =КОРЕНЬ(число).

Единственный и обязательный аргумент представляет собой положительное число, для которого функция вычисляет квадратный корень. Если аргумент имеет отрицательное значение, Excel вернет ошибку #ЧИСЛО!.

В качестве аргумента можно указывать конкретное значение либо ссылку на ячейку с числовым значением.

Рассмотрим примеры.

Функция вернула квадратный корень числа 36. Аргумент – определенное значение.

Аргумент функции – ссылка на ячейку с положительным значением 36.

Функция вернула ошибку, т.к. аргумент – ссылка на ячейку с отрицательным значением.

Функция ABS возвращает абсолютное значение числа -36. Ее использование позволило избежать ошибки при извлечении квадратного корня из отрицательного числа.

Функция извлекла квадратный корень от суммы 13 и значения ячейки C1.



Функция возведения в степень в Excel

Синтаксис функции: =СТЕПЕНЬ(значение; число). Оба аргумента обязательные.

Значение – любое вещественное числовое значение. Число – показатель степени, в которую нужно возвести заданное значение.

Рассмотрим примеры.

В ячейке C2 – результат возведения числа 10 в квадрат.».

Обратите внимание! Дробная степень пишется в скобках.

Выполнили ту же задачу, но с использованием функции СТЕПЕНЬ.

Извлекли корень девятой степени из значения ячейки h2.

Извлекли корень пятой степени из суммы числа 9 и значения ячейки h2.

Те же математические операции можно выполнить с помощью функции СТЕПЕНЬ:

Таким образом, возвести в степень и извлечь корень n-й степени в Excel можно с помощью одной функции.

Как написать число в степени

Для корректного отображения числа в степени при демонстрации файла или его печати, необходимо произвести ряд манипуляций:

  1. Щелкаем по ячейке с числом правой кнопкой мыши. Выбираем «Формат ячеек» (или нажмите CTRL+1).
  2. В открывшемся меню переходим на вкладку «Число». Задаем «Текстовый» формат. Текстовый формат для значения в ячейке можно также задать через панель инструментов («Главная» – «Число»). После установки текстового формата цифра в ячейке становится слева.
  3. Рядом с цифрой вводим в ячейку значение со знаком «минус».
  4. Выделяем только значение степени («-3»). Вызываем меню «Формат ячеек». Устанавливаем видоизменение «Надстрочный». И нажимаем ОК.

Получили корректное отображение числа 5 в -3 степени.

Корни или нули многочленов степени выше 2

13

Теорема о факторах

Основная теорема алгебры

Стратегия поиска корней

Теорема о целочисленном корне

Сопряженные пары

Доказательство теоремы о множителях

Доказательство теоремы о целочисленном корне

В ЭТОЙ ТЕМЕ мы увидим, как найти корни многочлена степени больше 2.Это будет зависеть от предыдущей темы: Синтетическое деление.

Мы видели в этой теме то, что называется теоремой о факторах.

Факторная теорема. x r является множителем полинома P ( x ) тогда и только тогда, когда r является корнем из P ( x ).

Это означает, что если многочлен можно разложить на множители, например, следующим образом:

P ( x ) = ( x — 1) ( x + 2) ( x + 3)

, то теорема говорит нам, что корни равны 1, −2 и −3.

И наоборот, если мы знаем, что корни многочлена равны −2, 1 и 5, то многочлен имеет следующие множители:

( x + 2) ( x — 1) ( x — 5).

Затем мы могли бы перемножить и узнать многочлен, имеющий эти три корня.

Ниже мы увидим, как доказать факторную теорему.

Задача 1.

a) Используйте факторную теорему, чтобы доказать: ( x + 1) множитель x 5 + 1.

Чтобы увидеть ответ, наведите указатель мыши на цветную область.
Чтобы закрыть ответ еще раз, нажмите «Обновить» («Reload»).
Сначала решите проблему сами!

−1 является корнем из x 5 + 1. Для, (−1) 5 + 1 = −1 + 1 = 0.
Следовательно, согласно теореме о множителях,
[ x — (- 1)] = ( x + 1) является множителем.

б) Используйте синтетическое деление, чтобы найти другой фактор.

Следовательно, x 5 + 1 = ( x + 1) ( x 4 x 3 + x 2 x + 1)

Следуя той же процедуре, мы можем доказать:

( x + a ) — коэффициент x 5 + a 5 ,

и полностью в целом:

( x + a ) является множителем x n + a n , где n является нечетным.

Основная теорема алгебры

Следующее называется основной теоремой алгебры:

Многочлен степени n имеет по крайней мере один корень, действительный или комплексный.

Это на первый взгляд простое утверждение позволяет сделать вывод:

Многочлен P ( x ) степени n имеет ровно n корней, действительных или комплексных.

Если старший коэффициент P ( x ) равен 1, то теорема о факторах позволяет нам сделать вывод:

P ( x ) = ( x r n ) ( x r n — 1 ). . . ( x r 2 ) ( x r 1 )

Следовательно, многочлен третьей степени, например, будет иметь три корня.А если все они настоящие, то его график будет выглядеть примерно так:

Ибо три корня — это три интерцепта x .

Примечание: Если представить, что график начинается слева от y — ось, тогда этот график начинается с ниже оси x . Почему? Потому что в любом полиноме главный член в конечном итоге будет доминировать. Если ведущий член положительный, а полином равен нечетным градусам, тогда, когда x является большим отрицательным числом , то есть далеко слева от начала координат, тогда нечетная степень отрицательного числа сам по себе отрицательный.График будет ниже оси x .

Что касается многочлена четвертой степени, то у него будет четыре корня. А если все они настоящие, то его график будет выглядеть примерно так:

Здесь крайний левый график — над осью x . Поскольку, когда полином равен , даже градусов (и старший коэффициент положительный), тогда четная степень отрицательного числа будет положительной .График будет выше оси x .

Пример 1. Напишите многочлен с целыми коэффициентами, имеющий следующие корни: −1, ¾.

Решение . Поскольку -1 — корень, то ( x + 1) — множитель. Что касается корня, то у нас было бы решение

x = 3
4
, что подразумевает
4 x = 3
4 x — 3 = 0

Факторы: (4 x — 3) ( x + 1).

Полином равен 4 x 2 + x — 3.

Задача 2. Определите многочлен, корни которого равны −1, 1, 2, и нарисуйте его график.

Факторы: ( x + 1) ( x — 1) ( x — 2). При умножении получается многочлен ( x 2 — 1) ( x — 2) =

x 3 — 2 x 2 x + 2.

Вот график:

Пересечение y — это постоянный член 2. В каждом полиноме перехват y — постоянный член, потому что постоянный член — это значение y , когда x = 0.

Задача 3. Определите многочлен с целыми коэффициентами, корни которых равны −½, −2, −2, и нарисуйте график.

Факторы: (2 x + 1) ( x + 2) 2 .При умножении многочлен равен (2 x + 1) ( x 2 + 4 x + 4) =

2 x 3 + 9 x 2 + 12 x + 4.

Вот график:

−2 — двойной корень. График не пересекает ось x .

Вопрос. Если r является корнем многочлена p ( x ), то после деления p ( x ) на x r , какой остаток следует ожидать?

0.Поскольку r является корнем, тогда x r — это коэффициент из p ( x ).

Задача 4. Является ли x = 2 корнем этого многочлена:

x 6 — 3 x 5 + 3 x 4 — 3 x 3 + 3 x 2 −3 x + 2?

Используйте синтетическое деление, чтобы разделить многочлен на x — 2, и посмотрите на остаток.

Остаток равен 0. 2 — корень многочлена.

Урок 12

Пример 2. Найдите три корня из

.

P ( x ) = x 3 -2 x 2 -9 x + 18,

с учетом того корень 3.

Решение. Поскольку 3 является корнем из P ( x ), то согласно теореме о множителях x — 3 является множителем. Следовательно, разделив P ( x ) на x — 3, мы можем найти другой квадратичный множитель.

У нас

x 3 — 2 x 2 — 9 x + 18 = ( x 2 + x — 6) ( x — 3)
= ( x — 2) ( x + 3) ( x — 3)

Три корня: 2, −3, 3.

Опять же, поскольку x — 3 является фактором P ( x ), остаток равен 0.

Задача 5. Нарисуйте график этого многочлена,

.

y = x 3 — 2 x 2 -5 x + 6,

, учитывая, что один корень равен -2.

Поскольку −2 является корнем, то ( x + 2) является множителем.Чтобы найти другой, квадратичный множитель, разделите многочлен на x + 2. Обратите внимание, что корень −2 помещается в поле:

У нас

x 3 — 2 x 2 -5 x + 6 = ( x 2 — 4 x + 3) ( x + 2)
= ( x — 1) ( x — 3) ( x + 2)

Три корня: 1, 3, −2.Вот график:

Стратегия поиска корней

Что же тогда представляет собой стратегия нахождения корней многочлена степени n > 2?

Нам нужно дать или угадать корень r . Затем мы можем разделить многочлен на x r и, следовательно, получить коэффициент полинома , который будет на один градус меньше. Если мы сможем найти корень этого фактора, мы сможем продолжить процесс, уменьшая степень каждый раз, пока не достигнем квадратичного значения, которое мы всегда сможем решить.

Вот теорема, которая поможет нам угадать корень.

Теорема о целочисленном корне. Если целое число является корнем многочлена, коэффициенты которого являются целыми числами, а старший коэффициент равен ± 1, то это целое число является коэффициентом постоянного члена.

Мы докажем это ниже.

Эта теорема о целочисленном корне является примером более общей теоремы о рациональном корне:

Если рациональное число об / с является корнем многочлена, коэффициенты которого являются целыми числами, то целое число r является множителем постоянного члена, а целое число s является множителем старшего коэффициента.

Пример 3. Каковы возможные целочисленные корни x 3 — 4 x 2 + 2 x + 4?

Ответ. Если есть целые корни, они будут множителями постоянного члена 4; а именно: ± 1, ± 2, ± 4.

Итак, 1 — это корень? Чтобы ответить, мы разделим многочлен на x — 1 и надеемся на остаток 0.

1 — 4 + 2 + 4 | 1
+ 1 — 3 — 1
———————————————— ————————————————— ——
1 — 3 — 1 + 3

Остаток не равен 0.1 не является корнем. Давайте попробуем −1:

1 — 4 + 2 + 4 | -1
— 1 + 5 — 7
———————————————— ————————————————— ——
1 — 5 + 7 — 3

Остаток снова не равен 0.Попробуем 2:

1 — 4 + 2 + 4 | 2
+ 2 — 4 — 4
———————————————— ————————————————— ——
1 — 2 — 2 + 0

Да! 2 — это корень.У нас

x 3 — 4 x 2 + 2 x + 4 = ( x 2 — 2 x — 2) ( x — 2)

Теперь мы можем найти корни квадратичного, завершив квадрат. Как мы обнаружили в теме 11:

.

x = 1 ±

Следовательно, три корня:

1+, 1 -, 2.

Проблема 6.

а) Каковы возможные целые корни этого многочлена?

x 3 — 2 x 2 — 3 x + 1

± 1. Это единственные факторы постоянного члена.

б) Имеет ли этот многочлен целые корни?

Нет, потому что ни 1, ни -1 не сделают этот многочлен равным 0. Синтетическое деление на оба ± 1 не дает остатка 0.

Задача 7. Разложите этот многочлен на произведение линейных множителей.

x 3 + 2 x 2 — 5 x — 6

Мы должны найти корни. Возможные целочисленные корни: ± 1, ± 2, ± 3, ± 6. Синтетическое деление показывает, что -1 — это корень.

Следовательно,

x 3 + 2 x 2 -5 x -6 = ( x + 1) ( x 2 + x — 6)
= ( x + 1) ( x + 3) ( x −2)

Сопряженные пары

Если иррациональное число a + является корнем, то его сопряжение a — также является корнем.(См. «Навыки алгебры», Урок 28.) И если комплексное число a + bi является корнем, то и его сопряженное число a bi .

Пример 4. Многочлен P ( x ) имеет следующие корни:

−2, 1 +, 5 и .

Какую наименьшую степень может иметь P ( x )?

Ответ .5. Ибо, поскольку 1 + — корень, то и сопряженный с ним 1 -. А поскольку 5 i является корнем, то же самое и сопряженное с ним, −5 i .

P ( x ) имеет как минимум эти 5 корней:

−2, 1 ±, ± 5 и .

Задача 8. Постройте многочлен со следующим корнем:

а) 2 +

Поскольку 2 + является корнем, то 2 — также. Следовательно, согласно теореме о сумме и произведении корней, они являются корнями x 2 — 4 x + 1.

Тема 10

б) 2 — 3 и

Поскольку 2 — 3 i — это корень, то 2 + 3 i — тоже. Опять же, согласно теореме суммы и произведения корней, они являются корнями x 2 — 4 x + 13.

См. Тему 10, пример 7.

Задача 9. Построить многочлен с корнями 1 и 5 i .

Поскольку 5 i является корнем, то также и его сопряженный элемент, −5 i . Они будут корнями квадратичного множителя многочлена. Сумма этих корней равна 0. Произведение равно 25. Следовательно, квадратичный коэффициент равен ( x 2 + 25).

Далее, поскольку 1 — это корень, тогда ( x — 1) является множителем. Следовательно, многочлен

( x — 1) ( x 2 + 25) = x 3 x 2 + 25 x — 25.

Задача 10. Пусть f ( x ) = x 5 + x 4 + x 3 + x 2 — 12 x — 12. Один корень равно, а другой — −2 i .

Если f ( x ) имеет целочисленные корни, сколько их может быть?

Один. Это многочлен 5-й степени, имеющий 5 корней. Два и -.И два — 2 i и −2 i .

Задача 11. Может ли многочлен 5-й степени иметь 2 действительных корня и 3 мнимых корня?

Нет это не так. Поскольку мнимые корни всегда попадают в пары, то, если есть какие-то мнимые корни, их всегда будет четное число.

Рассмотрим график многочлена 5-й степени с положительным старшим членом. Когда x — большое отрицательное число, график находится ниже оси x .Когда x — большое положительное число, оно находится выше оси x . Следовательно, график должен хотя бы один раз пересечь ось x . Теперь вы можете нарисовать график так, чтобы он пересекал ось x ровно дважды? Нет, ты не можешь. Многочлен нечетной степени должен иметь нечетное число действительных корней.

Доказательство теоремы о множителях

x r является множителем полинома P ( x )
тогда и только тогда, когда
r является корнем P ( x ).

Во-первых, если ( x r ) является множителем P ( x ), тогда P ( r ) будет иметь множитель ( r r ), который равно 0. Это сделает P ( r ) = 0. Это означает, что r является корнем.

И наоборот, если r является корнем из P ( x ), то P ( r ) = 0. Но согласно теореме об остатках P ( r ) = 0 означает что после деления P ( x ) на x r , остаток равен 0. x r , следовательно, это коэффициент P ( x ).

Это то, что мы хотели доказать.

Доказательство теоремы о целочисленном корне

Если целое число является корнем многочлена, коэффициенты которого являются целыми числами, а старший коэффициент равен ± 1, то это целое число является коэффициентом и постоянного члена.

Пусть целое число r будет корнем этого многочлена:

P ( x ) = ± x n + a n −1 x n −1 + a n −2 x n −2 +.. . + a 2 x 2 + a 1 x + a 0 ,

, где и — целые числа. Тогда, поскольку r является корнем,

P ( r ) = ± r n + a n −1 r n −1 + a n −2 r n −2 +.. . + a 2 r 2 + a 1 r + a 0 = 0,

Преобразуйте постоянный член в 0 и множите на из оставшихся членов:

r r n −1 + a n −1 r n −2 +. + a 2 r + a 1 ) = — a 0

Теперь все и являются целыми числами; поэтому выражение в скобках является целым числом, которое для удобства назовем — q:

r (−q) = — a 0 ,

или,

rq = а 0 .

Таким образом, постоянный член a 0 может быть разложен на множители как rq , если r и q являются целыми числами. Таким образом, в этих условиях r является множителем постоянного члена.

Это то, что мы хотели доказать.

Следующая тема: Множественные корни

Содержание | Дом


Сделайте пожертвование, чтобы TheMathPage оставалась в сети.
Даже 1 доллар поможет.


Авторские права © 2021 Лоуренс Спектор

Вопросы или комментарии?

Эл. Почта: [email protected]


Как найти корни многочлена

Обновлено 8 декабря 2020 г.

Лиза Мэлони

Корни многочлена также называются его нулями, потому что корнями являются значения x , при которых функция равно нулю. Когда дело доходит до фактического поиска корней, в вашем распоряжении несколько методов; Факторинг — это метод, который вы будете использовать чаще всего, хотя графическое отображение также может быть полезным.

Сколько корней?

Исследуйте член наивысшей степени полинома, то есть член с наивысшим показателем. Этот показатель показывает, сколько корней будет иметь многочлен. Итак, если наивысший показатель в вашем полиноме равен 2, он будет иметь два корня; если наивысший показатель равен 3, у него будет три корня; и так далее.

Найти корни путем разложения на множители: пример 1

Самый универсальный способ нахождения корней — это максимально возможное разложение вашего полинома на множители, а затем установка каждого члена равным нулю.В этом будет больше смысла, если вы рассмотрите несколько примеров. Рассмотрим простой многочлен x 2 — 4 x:

    Краткий анализ показывает, что вы можете разложить x на множители из обоих членов полинома, что дает вам:

    x (x — 4)

    Обнулить каждый член. Это означает решение двух уравнений:

    x = 0

    — это первый член, установленный в ноль, и

    x — 4 = 0

    — второй член, установленный в ноль.

    У вас уже есть решение для первого члена. Если x = 0, то все выражение равно нулю. Итак, x = 0 — это один из корней или нулей многочлена.

    Теперь рассмотрим второй член и решим относительно x . Если вы добавите 4 к обеим сторонам, вы получите:

    x — 4 + 4 = 0 + 4

    x = 4

    Итак, если x = 4, то второй множитель равен нулю, что означает весь многочлен тоже равен нулю.

    Поскольку исходный многочлен был второй степени (старший показатель был равен двум), вы знаете, что у этого многочлена есть только два возможных корня. Вы уже нашли их оба, поэтому все, что вам нужно сделать, это перечислить их:

    x = 0, x = 4

Найти корни по факторингу: Пример 2

Вот еще один пример того, как найти корни путем факторизации, попутно используя причудливую алгебру. Рассмотрим полином x 4 -16.2 + 4)

А теперь пора найти нули. Быстро становится ясно, что если x = 2, первый множитель будет равен нулю, и, таким образом, все выражение будет равно нулю.

Точно так же, если x = −2, второй множитель будет равен нулю, и, следовательно, будет все выражение.

Таким образом, x = 2 и x = −2 являются нулями или корнями этого многочлена.

А как насчет последнего семестра? Поскольку у него показатель степени «2», он должен иметь два корня.Но вы не можете разложить это выражение на множители, используя реальные числа, к которым вы привыкли. Вам придется использовать очень продвинутую математическую концепцию, называемую мнимыми числами или, если хотите, комплексными числами. Это далеко выходит за рамки вашей нынешней математической практики, поэтому пока достаточно отметить, что у вас есть два реальных корня (2 и −2) и два мнимых корня, которые вы оставите неопределенными.

Найдите корни с помощью графика

Вы также можете найти или, по крайней мере, оценить корни с помощью графика.Каждый корень представляет собой точку, в которой график функции пересекает ось x . Итак, если вы построите линию, а затем обратите внимание на координаты x , где линия пересекает ось x , вы можете вставить оценочные значения x этих точек в свое уравнение и проверить, чтобы увидеть если вы их правильно поняли.

Рассмотрим первый пример, с которым вы работали, для многочлена x 2 — 4 x . Если вы нарисуете его внимательно, вы увидите, что линия пересекает ось x в точках x = 0 и x = 4.2-4 (4) = 0

, поэтому x = 4 также является допустимым нулем или корнем для этого многочлена. А поскольку многочлен имел степень 2, вы знаете, что можете перестать искать два корня.

Как решать полиномиальные уравнения

Как решать полиномиальные уравнения

Авторские права © 20022020 Стэн Браун

Сводка: В алгебре вы тратите много времени на решение многочлена уравнения или факторизации многочленов (что одно и то же). Было бы легко потеряться во всех техниках, но эта статья связывает их все вместе в единое целое.


Генеральный план

Убедитесь, что вас не смущает терминология. Все это то же:

  • Решение полиномиального уравнения p ( x ) = 0
  • Нахождение корней полиномиального уравнения p ( x ) = 0
  • Нахождение нулей полиномиальной функции p ( x )
  • Факторизация полиномиальной функции p ( x )

Есть коэффициент для каждого корня, и наоборот. ( x r ) является фактором тогда и только тогда, когда r является корнем. Это Теорема о факторах : поиск корней или факторов по сути то же самое. (Основное различие заключается в том, как вы относитесь к постоянный коэффициент.)

Точное или Приблизительное?

Чаще всего, когда мы говорим о решении уравнения или факторизации многочлен, мы имеем в виду точное (или аналитическое) решение . В другой тип, приближенное (или числовое) решение , всегда возможно, а иногда и единственная возможность.

Когда найдешь, точное решение лучше . Вы всегда можете найти численное приближение к точному решению, но пойти другим путем гораздо труднее. Эта страница тратит больше всего своего времени на методы точных решений, но также расскажет, что нужно делать, когда аналитические методы терпят неудачу.

Шаг за шагом

Как найти множители или нули многочлена (или корни полиномиального уравнения)? По сути, вы сточите . Каждый раз вы вычеркиваете множитель или корень из многочлена, у вас остается полином на одну степень проще.Используйте этот новый уменьшенный полином, чтобы найти оставшиеся факторы или корни.

На любом этапе процедуры, если вы попадете в кубическое или четвертое уравнение (степень 3 или 4), у вас есть выбор продолжения факторинга или использования кубические или четвертичные формулы. Этих формул много работы, поэтому большинство людей предпочитают продолжать факторинг.

Следуйте этой процедуре шаг за шагом:

  1. При решении уравнения запишите его в стандартную форму с 0 с одной стороны и упрощают .[ подробности ]
  2. Знайте , сколько корней ожидать. [ подробности ]
  3. Если у вас есть линейное или квадратное уравнение (степень 1 или 2), решите осмотром или по формуле корней квадратного уравнения. [ подробности ]
    Затем переходите к шагу 7.
  4. Найдите один рациональный множитель или корень. Это самая сложная часть, но есть много методов, которые могут вам помочь. [ подробности ]
    Если вы можете найти фактор или корень, перейдите к шагу 5 ниже; если не можете, переходите к шагу 6.
  5. Разделите на множитель . Это оставляет вас с новым приведенный многочлен , степень которого на 1 меньше. [ подробности ]
    Для остальной части задачи вы будете работать с уменьшенным многочлен, а не оригинал. Продолжите с шага 3.
  6. Если вы не можете найти множитель или корень , обратитесь к численные методы. [ подробности ]
    Затем переходите к шагу 7.
  7. Если это уравнение нужно было решить, запишите корни . Если это был многочлен для разложения на множители, запишите его в факторизованной форме , включая любые постоянные факторы, которые вы вывели на шаге 1.

Это пример алгоритма , набор шагов что приведет к желаемому результату за конечное количество операций. Это итеративная стратегия , потому что средние шаги повторять столько, сколько необходимо.

Кубические и четвертые формулы

Приведенные здесь методы находят рациональный корень и использовать синтетическое деление проще всего. Но если вы не можете найти рациональный корень, есть специальные методы для кубические уравнения (степень 3) и уравнения четвертой степени (степень 4), оба в Mathworld.Альтернативный подход предоставляется Дик Никаллс в PDF для кубический а также четвертичная уравнения.


Шаг 1. Стандартная форма и упрощение

К сожалению, это легко не заметить. Если у вас есть полиномиальное уравнение , отложите все члены в одну сторону и 0 с другой. И будь то проблема факторинга или уравнение, которое нужно решить, положите ваш многочлен в стандартной форме, от до наименьшей степени .

Например, вы не можете решить это уравнение в такой форме:

x + 6 x + 12 x = −8

Вы должны изменить его на эту форму:

x + 6 x + 12 x + 8 = 0

Также убедитесь, что вы упростили, исключив любые общие факторы .Это может включать в себя вычитание −1 так что наивысшая степень имеет положительный коэффициент. Пример: коэффициент

7-6 x -15 x — 2 x

начнем с его стандартной формы:

−2 x — 15 x — 6 x + 7

, а затем вычтите −1

— (2 x + 15 x + 6 x — 7) или же (−1) (2 x + 15 x + 6 x -7)

Если вы решаете уравнение, вы можете выбросить любое общий постоянный множитель.Но если вы факторизуете многочлен, вы должны сохранить общий множитель .

Пример: решить 8 x + 16 x + 8 = 0, можно разделите левую и правую на общий множитель 8. Уравнение x + 2 x + 1 = 0 имеет те же корни, что и исходное уравнение .

Пример: Фактор 8 x + 16 x + 8, вы узнаете общий множитель 8 и перепишем многочлен в виде 8 ( x + 2 x + 1), что является идентичен исходному многочлену .(Хотя это правда, что вы сосредоточит ваши дальнейшие усилия по факторингу на x + 2 x + 1, это будет ошибкой написать, что исходный многочлен равен x + 2 x + 1.)

Ваш общий фактор может быть дробь, потому что вы должны вычесть любые дроби, чтобы многочлен имеет целочисленных коэффициентов .

Пример: решить (1/3) x + (3/4) x — (1/2) x + 5/6 = 0, вы узнаете общий множитель 1/12 и разделите обе стороны на 1/12.Это в точности то же самое, что и распознавание и умножение на наименьший общий знаменатель из 12. В любом случае вы получите 4 x + 9 x — 6 x + 10 = 0, которое имеет те же корни, что и исходное уравнение .

Пример: Фактор (1/3) x + (3/4) x — (1/2) x + 5/6, вы узнаете общий множитель 1/12 (или наименьший общий знаменатель 12) и вычитаем 1/12. Ты получаешь (1/12) (4 x + 9 x — 6 x + 10), что идентично исходному многочлену .

Шаг 2. Сколько корней?

Многочлен степени n будет иметь n корней, некоторые из которых могут быть множественные корни.

Как узнать, что это правда? В Основная теорема алгебры говорит вам, что многочлен имеет хотя бы один корень. Теорема о факторах говорит вам, что если r является корнем, тогда ( x r ) является фактором. Но если разделить многочлен степени n на множитель ( x r ), степень которого равна 1, вы получите многочлен степени n −1.Неоднократно применяя Фундаментальный Теорема и теорема о множителях дают вам n корней и n факторов.

Правило знаков Декарта

Правило знаков Декарта может сказать вам , сколько положительных и сколько отрицательных действительных нулей многочлен. Это большое трудосберегающее устройство, особенно когда вы решаете, какой возможные рациональные корни, которые нужно искать.

Чтобы применить Правило знаков Декарта, вам необходимо понимать термин изменение знака .Когда многочлен расположен в стандартная форма, вариант Знак возникает, когда знак коэффициента отличается от знака предыдущего коэффициента. (Нулевой коэффициент игнорируется.) Для пример,

p ( x ) = x 5 — 2 x 3 + 2 x 2 — 3 x + 12

имеет четыре варианта знака.

Правило знаков Декарта:

  • Число положительных корней из p ( x ) = 0 либо равно количество вариаций знака p ( x ), или меньше, чем на четное номер.
  • Число отрицательных корней из p ( x ) = 0 либо равно количество вариаций знака p (- x ), или меньше на четное номер.

Пример: рассмотрим p ( x ) выше. Поскольку у него четыре варианта в знаке должно быть либо четыре положительных корня, либо два положительных корня, или нет положительных корней.

Теперь сформируйте p (- x ), заменив x на (- x ) в выше:

p (- x ) = (- x ) 5 — 2 (- x ) 3 + 2 (- x ) 2 — 3 (- x ) + 12

p (- x ) = — x 5 + 2 x 3 + 2 x 2 + 3 x + 12

p (- x ) имеет один вариант знака, поэтому оригинал p ( x ) имеет один негатив корень.Поскольку вы знаете, что p ( x ) должен иметь отрицательный корень, но он может или может не иметь положительных корней, сначала ищите отрицательные корнеплоды.

p ( x ) — полином пятой степени, поэтому он должен иметь пять нулей. Поскольку x не является фактором, вы знаете, что x = 0 не является нуль полинома. (Для полинома с действительными коэффициентами, например в этом случае комплексные корни встречаются парами.) Следовательно, есть три возможности:

количество нулей
, которые равны
положительные отрицательные комплексные
нереальные
первая возможность 4 12 90272 90272 9027 2 1 2
третий вариант 0 1 4

Сложные корни

Если полином имеет действительных коэффициентов , то либо все корни настоящие или есть четное число невещественных комплексных корней в сопряженных парах .

Например, если 5 + 2i является нулем многочлена с вещественными коэффициентов, то 5−2i также должен быть нулем этого многочлена. Также верно и то, что если ( x −5−2i) является множителем, то ( x −5 + 2i) также является фактором.

Почему это правда? Потому что, когда у вас есть фактор с воображаемым часть и умножьте ее на комплексное сопряжение, вы получите реальную результат:

( x −5−2i) ( x −5 + 2i) = x −10 x + 25−4i = x −10 x +29

Если ( x −5−2i) было фактором, но ( x −5 + 2i) не было, тогда многочлен будет иметь воображение в его коэффициентах, независимо от других факторов возможно.Если многочлен имеет только действительные коэффициентов, то любые комплексные корни должны входить в сопряженные пары.

Иррациональные корни

По аналогичным причинам, если многочлен имеет рациональных коэффициентов то иррациональные корни, содержащие квадратный корни встречаются (если вообще встречаются) в сопряженных парах. Если ( x −2 + √3) является множителем многочлена с рациональными коэффициентов, то ( x −2 − √3) также должны быть фактор. Чтобы понять почему, вспомните, как вы рационализируете бином. знаменатель; или просто проверьте, что происходит, когда вы умножаете эти два факторы.(1/3) и два сложные корни.

Интересная проблема, есть ли иррациональность с четными корнями порядка ≥4 также должны встречаться в сопряженных пары. У меня нет немедленного ответа. Я работаю над пруф, как я успеваю.

Множественные корни

Когда данный множитель ( x r ) встречается в полиноме m раз, r составляет называется кратным корнем или корнем кратностью м .

  • Если кратность м — четное число, график касается Ось x при x = r , но не пересекает ее.
  • Если кратность м — нечетное число, график пересекает Ось x при x = r . Если кратность 3, 5, 7 и т. Д., График горизонтально в точке пересечения оси.

Примеры: сравните эти два многочлена и их графики:

f ( x ) = ( x −1) ( x −4) 2 = x 3 — 9 x 2 + 24 x — 16

г ( x ) = ( x −1) 3 ( x −4) 2 = x 5 — 11 x 4 + 43 x 3 — 73 x 2 + 56 x — 16

Эти многочлены имеют одинаковые нули, но корень 1 встречается с разной кратностью.Посмотрите на графики:

Оба полинома имеют нули только в точках 1 и 4. f ( x ) имеет степень 3, что означает три корня. Вы видите из факторов, что 1 является корнем кратность 1 и 4 является корнем из кратности 2. Следовательно, граф пересекает ось x = 1 (но не горизонтально) и касается x = 4 без пересечения.

Напротив, г ( x ) имеет степень 5. ( г ( x ) = f ( x ) раз ( x −1) 2 .) Из пяти корней 1 встречается с кратность 3: график пересекает ось при x = 1 и является горизонтальным там; 4 встречается с кратностью 2, и график касается ось x = 4 без пересечения.


Шаг 3. Квадратичные множители

Когда у вас есть квадратичные множители (Ax + Bx + C), он может или не может можно будет их дополнительно проанализировать.

Иногда вы можете просто увидеть факторы, как в случае с x x −6 = ( x +2) ( x −3).В других случаях не так очевидно, квадратичный можно разложить на множители. Вот когда квадратная формула (показан справа) ваш друг.

Например, предположим, что у вас есть коэффициент 12 x x −35. Можно ли это еще раз проанализировать? Судом и ошибка вам придется перепробовать много комбинаций! Вместо этого используйте факт что факторов соответствуют корням , и примените формулу к найти корни 12 x x −35 = 0, например:

x = [- (- 1) √1 — 4 (12) (- 35)] / 2 (12)

x = [1 √1681] / 24

√1681 = 41, следовательно,

x = [1 41] / 24

x = 42/24 или -40/24

x = 7/4 или -5/3

Если 7/4 и −5/3 являются корнями, то ( x −7/4) и ( x +5/3) факторы.Следовательно,

12 x x −35 = (4 x −7) (3 x +5)

А как насчет x −5 x +7? Этот выглядит как лучший, но как ты можешь быть уверен? Снова примените формулу:

x = [- (- 5) √25 — 4 (1) (7)] / 2 (1)

x = [5 √ − 3] / 2

Что с этим делать, зависит от исходной проблемы. Если это должен был разложить на множители реалы, тогда x −5 x +7 — простое число.Но если этот фактор был частью уравнения, и вы должны были найти все сложные корни, у вас их два:

x = 5/2 + (√3 / 2) i, x = 5/2 — (√3 / 2) i

Поскольку исходное уравнение имело действительные коэффициенты, эти сложные корни встречаются в сопряженной паре.


Шаг 4. Найдите один фактор или корень

Этот шаг является сердцем факторизации многочлена или решения полиномиальное уравнение. Есть много методов, которые могут вам помочь найти фактор.

Иногда можно найти факторы путем осмотра (см. Первые два следующие разделы). Это отличный способ быстрого доступа, поэтому проверьте легкие факторы, прежде чем начинать более напряженные методы.

Мономиальные множители

Всегда начинайте с поиска любых мономиальных множителей, которые вы видите. Например, если ваша функция

f ( x ) = 4 x 6 + 12 x 5 + 12 x 4 + 4 x 3

, вы должны немедленно разложить его на

f ( x ) = 4 x 3 ( x 3 + 3 x 2 + 3 x + 1)

Получение числа 4 упрощает оставшиеся числа, x 3 дает вам корень x = 0 (с кратностью 3), и теперь у вас есть только кубический многочлен (степени 3) вместо sextic (степень 6).Фактически, теперь вы должны распознать эту кубику как особый продукт, идеальный куб ( x +1) 3 .

Когда вы вычитаете множитель общей переменной, убедитесь, что вы помните об этом в конце, когда перечисляете фактор или корни. x +3 x +3 x +1 = 0 имеет определенные корни, но x ( x +3 x +3 x +1) = 0 имеет те же корни и также корень с размером x = 0 (с кратностью 3).

Особые продукты

Будьте внимательны к применению Special Products .Если вы сможете применить их, ваша задача станет намного проще. Специальный Продукты:

  • идеальный квадрат (2 формы): A 2 A B + B = ( A B )
  • сумма квадратов: A + B не может быть разложено на множители в целом (для исключительных случаев см. Как разложить на множители сумму квадратов)
  • разность квадратов: A B = ( A + B ) ( A B )
  • идеальный куб (2 формы): A 3 A B + 3 A B B = ( A B )
  • сумма кубов: A + B = ( A + B ) ( A A B + B )
  • разность кубов: A B = ( A B ) ( A + A B + B )

Выражения для суммы или разности двух кубов выглядят так: хотя они должны учитывать дополнительные факторы, но они этого не делают. A A B + B является простым над реалами.

Рассмотрим

p ( x ) = 27 x — 64

Вы должны узнать это как

p ( x ) = (3 x ) — 4

Вы умеете множить разницу двух кубов:

p ( x ) = (3 x −4) (9 x +12 x +16)

Бинго! Как только вы дойдете до квадратичной, вы можете применить Квадратичная формула, и все готово.

Вот другой пример:

q ( x ) = x 6 + 16 x 3 + 64

Это просто идеальный квадратный трехчлен, но вместо этого в x 3 размером x . Вы учитываете это точно так же:

q ( x ) = ( x 3 ) 2 + 2 (8) ( x 3 ) + 8 2

q ( x ) = ( x 3 + 8) 2

И вы можете легко разложить на множители ( x 3 +8) 2 как ( x +2) 2 ( x 2 −2 x +4) 2 .

Рациональные корни

Предполагая, что вы уже учли простые мономиальные факторы и специальные продукты, что вы будете делать, если у вас все еще есть многочлен степени 3 или выше?

Ответ — Rational Root Test . Он может показать вам некоторые корни кандидатов когда вы не видите, как разложить многочлен на множители, как показано ниже.

Рассмотрим многочлен стандартной формы, записанный с высшей степени до самого низкого и с целыми коэффициентами :

f ( x ) = a n x n +… + a o

Теорема рационального корня говорит вам, что , если многочлен имеет рациональный нуль , затем , это должна быть дробь p / q , где p — коэффициент задней константы a o и q — множитель ведущего коэффициента a n .

Пример:

p ( x ) = 2 x 4 — 11 x 3 — 6 x 2 + 64 x + 32

Коэффициенты старшего коэффициента (2) равны 2 и 1.В коэффициенты постоянного члена (32) равны 1, 2, 4, 8, 16 и 32. Следовательно, возможные рациональные нули: 1, 2, 4, 8, 16 или 32 разделить на 2 или 1:

любой из 1/2, 1/1, 2/2, 2/1, 4/2, 4/1, 8/2, 8/1, 16/2, 16/1, 32/2, 32/1

уменьшено: любое из, 1, 2, 4, 8, 16, 32

Что мы имеем в виду, когда говорим, что это список всех возможных рациональных корней ? Мы имеем в виду, что никакое другое рациональное число, как или 32/7, может быть нулем этого конкретного многочлена.

Внимание : Не делайте Rational Root Test больше, чем есть.Это не означает, что рациональные числа являются корнями , просто что никакие другие рациональные числа не могут быть корнями. И это не говорит вы что-нибудь о том, есть ли какие-то иррациональные или даже сложные корни существовать. Rational Root Test — это только отправная точка.

Предположим, у вас есть многочлен с нецелыми коэффициентами. Вы застряли? Нет, вы можете исключить наименее распространенные знаменатель (LCD) и получите многочлен с целыми коэффициентами, которые способ. Пример:

(1/2) x — (3/2) x + (2/3) x — 1/2

ЖК-дисплей 1/6.Вынося за скобки 1/6 получаем многочлен

.

(1/6) (3 x — 9 x + 4 x — 3)

Эти две формы эквивалентны, и поэтому имеют одинаковые корнеплоды. Но вы не можете применить Rational Root Test к первой форме, только ко второму. Тест говорит вам, что единственно возможное рациональное корни — любые из 1/3, 1, 3.

После того, как вы определили возможных рациональных нулей, как вы можете их проверить? Метод грубой силы заключался бы в том, чтобы взять каждый возможное значение и замените его на x в полиноме: если результат равен нулю, тогда это число является корнем.Но есть лучше способ.

Используйте Synthetic Division, чтобы узнать, кандидат делает полином равным нулю. Это лучше на троих причины. Во-первых, это проще в вычислительном отношении, потому что вам не нужно вычислить высшие степени чисел. Во-вторых, в то же время он сообщает независимо от того, является ли данное число корнем, он производит сокращенный многочлен , который вы будете использовать, чтобы найти оставшийся корнеплоды. Наконец, результаты синтетического деления могут дать вам верхняя или нижняя граница, даже если число тестирование оказывается не рутом.

Иногда правило знаков Декарта может поможет вам в дальнейшем выявить возможные рациональные корни. Например, Rational Root Test сообщает, что если

q ( x ) = 2 x 4 + 13 x 3 + 20 x 2 + 28 x + 8

имеет какие-либо рациональные корни, они должны происходить из списка любой из, 1, 2, 4, 8. Но не начинайте с замены или синтетическое разделение. Поскольку нет изменений знака, нет положительные корни.Есть ли отрицательные корни?

q (- x ) = 2 x 4 -13 x 3 + 20 x 2 -28 x + 8

имеет четыре смены знака. Следовательно, может быть целых четыре отрицательные корни. (Также может быть два отрицательных корня или ни одного.) Нет гарантии, что какой-либо из корней является рациональным, но любой корень рациональное должно происходить из списка -, −1, −2, −4, −8.

(Если у вас графического калькулятора, вы можете предварительно просмотреть рациональные корни, построив график полином и увидеть, где он, кажется, пересекает ось x .Но ты по-прежнему необходимо проверить корень алгебраически, чтобы увидеть, что f ( x ) равно ровно 0, а не почти 0.)

Помните, что Rational Root Test гарантирует нахождение всех рациональных корней. Но он полностью упустит настоящие корни, которых нет. рациональные, как корни x −2 = 0, которые √2, или корни x + 4 = 0, которые 2i.

Наконец, помните, что Rational Root Test работает, только если все коэффициенты — целые числа.Посмотрите еще раз на эту функцию, которая на графике справа:

p ( x ) = 2 x 4 — 11 x 3 — 6 x 2 + 64 x + 32

Теорема о рациональном корне говорит вам, что единственно возможный рациональный нули — это 1, 2, 4, 8, 16, 32. Но предположим, что вы вычтите 2 (как я когда-то сделал в классе), написав эквивалент функция

p ( x ) = 2 ( x 4 — (11/2) x 3 -3 x 2 + 32 x + 16)

Эта функция аналогична предыдущей, но вы не можете дольше применять Rational Root Test, потому что коэффициенты не целые числа.По сути — это ноль p ( x ), но это не так. появляются, когда я (незаконно) применил Rational Root Test к вторая форма. Моя ошибка заключалась в том, что я забыл, что применима теорема о рациональном корне. только когда все коэффициенты многочлена равны целые числа.

Графические подсказки

Построение графика функции вручную или с помощью графика Вы можете понять, где находятся корни, примерно, и сколько существует настоящих корней.

Пример: Если Rational Root Test говорит вам, что 2 возможных рациональных корня, вы можете посмотреть на график, чтобы увидеть, пересекает ли он (или касается) оси x в точках 2 или −2.Если да, используйте синтетическое деление, чтобы убедитесь, что предполагаемый корень на самом деле является корнем. Да ты всегда нужно проверить по графику, вы никогда не можете быть уверены является ли точка пересечения на ваш возможный рациональный корень или всего рядом с он.

Границы корней

Некоторые методы не сообщают вам конкретное значение корня, но скорее, что корень существует между двумя значениями или что все корни меньше определенного числа больше определенного числа. Этот помогает сузить область поиска.

Теорема о промежуточном значении

Эта теорема говорит вам, что если график многочлена находится выше ось x для одного значения x и ниже оси x для другое значение x , оно должно пересекать ось x где-то посередине. (Если вы можете построить график функции, пересечения обычно будет очевидным.)

Пример:

p ( x ) = 3 x + 4 x — 20 x −32

Рациональные корни (если есть) должны быть из списка любой из 1/3, 2/3, 1, 4/3, 2, 8/3, 4, 16/3, 8, 32/3, 16, 32.Естественно, сначала вы посмотрите на целые числа, потому что арифметика Полегче. Пробуя синтетическое деление, вы найти p (1) = −45, p (2) = −22 и p (4) = 144. Поскольку p (2) и p (4) имеют противоположные знаки, вы знайте, что график пересекает ось между x = 2 и x = 4, поэтому хотя бы один корень между этими числами. Другими словами, либо 8/3 — это корень или корень от 2 до 4 иррациональны. (По факту, синтетическое деление показывает, что 8/3 — это корень.)

Теорема о промежуточном значении может сказать вам, где находится root, но он не может сказать вам, где нет root. Например, считать

q ( x ) = 4 x — 16 x + 15

q (1) и q (3) оба положительны, но это вам не говорит может ли график касаться или пересекать ось между ними. (Это на самом деле дважды пересекает ось, при x = 3/2 и x = 5/2.)

Верхняя и нижняя границы

Одним из побочных эффектов синтетического деления является что даже если число, которое вы тестируете, окажется не корневым, оно может сказать вам, что все корни меньше или больше этого номер:

  • Если вы выполните синтетическое деление на положительное число , a , и каждое число в нижней строке положительное или нулевое, тогда a — это верхняя граница для корней, что означает, что все действительные корни ≤ a .
  • Если вы делаете синтетическое деление на отрицательное число b , а числа в нижнем ряду чередуются знак, тогда b — это нижняя граница для корней, что означает, что все действительные корни ≥ b .

    Что делать, если нижняя строка содержит нули? Более полный Утверждение состоит в том, что чередующихся неотрицательных и неположительных знаков , после синтетического деления на отрицательное число показать нижнюю границу корень. Следующие два примера поясняют это.

(Кстати, правило для нижних оценок следует из правила для верхних оценок. Нижние пределы корней p ( x ) равны верхним пределам корни p (- x ), и деление на (- x + r ) такое же, как деление на — ( x r ).)

Пример:

q ( x ) = x 3 + 2 x 2 — 3 x — 4

Использование Rational Root Тест, вы определяете единственные возможные рациональные корни как 4, 2 и 1.Вы решаете попробовать −2 как возможный корень, и вы тестируете его с синтетическим делением:

        -2 | 1 2 -3-4
            | -2 0 6
            | ------------------
               1 0–3 2 

−2 не является корнем уравнения f ( x ) = 0. В третьей строке чередуются знаки, и вы делили на отрицательное число; однако этот ноль все портит. Напомним, что у вас есть нижняя граница, только если знаки в нижнем ряду чередовать неположительный и неотрицательный.1 положительный (неотрицательный), и 0 может считаться неположительным, но −3 не считается неотрицательным. Чередование битая, а ты не знаешь есть ли корни меньше -2. (Фактически, графический или численные методы покажут корень около -2,5.) Следовательно, вам нужно попробовать наименьший возможный рациональный корень, −4:

.
        -4 | 1 2 -3-4
            | -4 8-20
            | ------------------
               1–2 5–24 

Здесь знаки чередуются; поэтому вы знаете, что нет корни ниже −4.(Остаток −24 показывает, что −4 сам по себе не является корнем.)

Вот другой пример:

r ( x ) = x + 3 x — 3

Rational Root Test сообщает вам что возможные рациональные корни — 1 и 3. С синтетическим деление на −3:

        -3 | 1 3 0-3
            | -3 0 0
            | ------------------
               1 0 0-3 

−3 не является корнем, но знаки здесь чередуются, так как первый 0 считается неположительным, а второй — неотрицательным.Следовательно, −3 — это нижняя граница корней, а это означает, что уравнение не имеет вещественных корней ниже −3.

Коэффициенты и корни

Существует интересная взаимосвязь между коэффициентами многочлен и его нули. Я упоминаю об этом в последнюю очередь, потому что это больше подходит для формирования многочлена, который имеет нули с желаемыми свойствами, вместо нахождения нулей существующего многочлена. Однако если вы знать все корни многочлена, кроме одного или двух, вы можете легко использовать это техника, чтобы найти оставшийся корень.

Рассмотрим многочлен

f ( x ) = a n x n + a n −1 x n −1 + a n −2 x n −2 + … + a 2 x 2 + a 1 x + а или

Существуют следующие отношения:

  • a n −1 a n = сумма всех корней
  • + a n − 2 a n = сумма произведений корней взято по два за раз
  • a n −3 a n = сумма произведений корней взято по три за раз
  • и так далее, пока
  • (-1) n a 0 a n = произведение всех корней

Пример: f ( x ) = x 3 — 6 x 2 — 7 x — 8 имеет степень 3 и, следовательно, не более трех реальных нулей.Если записываем действительные нули как r 1 , r 2 , r 3 , то сумма корней равна r 1 + r 2 + r 3 = — (- 6) = 6; в сумма произведений корней, взятых по два за раз, равна r 1 r 2 + r 1 r 3 + r 2 r 3 = −7, а произведение корней равно r 1 r 2 r 3 = (-1) 3 (-8) = 8.

Пример: Учитывая, что многочлен

г ( x ) = x 5 -11 x 4 + 43 x 3 -73 x 2 + 56 x — 16

имеет тройной корень x = 1, найдите два других корня.

Решение: Пусть два других корня будут c и d . Тогда вы знаете, что сумма всех корней равна 1 + 1 + 1 + c + d = — (- 11) = 11, или c + d = 8.Ты также знайте, что продукт всех корней 111 c d = (−1) 5 (−16) = 16, или c d = 16. c + d = 8, c d = 16; поэтому c = d = 4, поэтому оставшиеся корни представляют собой двойной корень с размером x = 4.

Дополнительные коэффициенты и корни

Есть еще несколько теорем о соотношении между коэффициентами и корнями. Статья в Википедии Свойства корней полиномов дает хорошее, хотя и несколько краткое резюме.


Шаг 5. Разделите на множитель

Помните, что r является корнем тогда и только тогда, когда x r является фактором; это факторная теорема. Так что если ты хочешь чтобы проверить, является ли r корнем, вы можете разделить многочлен на x r и посмотрите, выходит ли ровным (остаток от 0). Элизабет Стапель имеет хороший пример деления многочленов делением в столбик.

Но делать синтетическое деление проще и быстрее.Если твой синтетическое деление немного заржавело, вы можете взглянуть на Dr. Математика короткая Учебник по синтетическому дивизиону; если вам нужен более длинный учебник, Элизабет Стейплс Синтетический дивизион отличный. (У доктора Мата также есть страница о почему работает Synthetic Division.)

Синтетическое подразделение также имеет некоторые побочные преимущества. Если вы подозреваете корень на самом деле является корнем, синтетическое деление дает вам приведенный многочлен . А иногда и тебе везет, и синтетическое деление показывает вам верхнюю или нижнюю привязаны к корням.

Вы можете использовать синтетическое деление при делении на бином вида x r для постоянной r . Если вы делите на x −3, вы проверяете, является ли 3 корнем, и вы синтетическое деление на 3 (не на −3). Если вы делите на x +11, вы тестируете является ли −11 корнем, и вы синтетически делите на −11 (не 11).

Пример:

p ( x ) = 4 x 4 — 35 x 2 — 9

Вы подозреваете, что x −3 может быть фактором, и проверяете это с помощью синтетическое деление, например:

        3 | 4 0-35 0-9
           | 12 36 3 9
           | --------------------
              4 12 1 3 0 

Поскольку остаток равен 0, вы знаете, что 3 является корнем p ( x ) = 0, а x −3 является множителем p ( x ).Но ты знаешь более. Поскольку 3 положительно и нижняя строка синтетического деления все положительные или нулевые, вы знаете, что все корни p ( x ) = 0 должно быть ≤ 3. И вы также знаете что

p ( x ) = ( x −3) (4 x 3 + 12 x 2 + x + 3)

4 x 3 + 12 x 2 + x + 3 — это приведенный многочлен .Все его факторы также коэффициентов исходного p ( x ), но его степень на единицу ниже , поэтому его с ним легче работать.


Шаг 6. Численные методы

Когда у вашего уравнения больше нет рациональных корней (или многочлен не имеет более рациональных множителей) можно перейти к числовым методы нахождения приблизительного значения иррациональных корней:

  • Статья в Википедии Алгоритм поиска корней имеет достойное резюме с указателями на конкретные методы.
  • Многие графические калькуляторы имеют Команда Root или Zero, которая поможет вам найти приблизительные корни. Например, на ТИ-83 или ТИ-84 вы график функцию, а затем выберите [2nd] [Calc] [zero].

Полный пример

Решить для всех сложных корней:

4 x + 15 x — 36 = 0

Шаг 1. Уравнение уже в стандартной форме, с только ноль с одной стороны и степень x от наибольшего до наименьшего.Там нет общих факторов.

Шаг 2. Так как уравнение имеет степень 3, будет 3 корнеплоды. Есть одна вариация знака, а от Правило знаков Декарта, которое, как вы знаете, должно быть одним положительным корнем. Изучите многочлен, заменив — x x :

−4 x — 15 x — 36

Знаков нет, значит, нет отрицательные корни. Следовательно, два других корня должны быть сложными, и конъюгаты друг друга.

Шаги 3 и 4. Возможные рациональные корни к сожалению, довольно много: любые из 1, 2, 3, 4, 6, 9, 12, 18, 36 делится на любое из 4, 2, 1. (перечислены только положительные корни, потому что вы уже определили, что для этого нет отрицательных корней уравнение.) Вы решаете сначала попробовать 1:

        1 | 4 0 15 -36
           | 4 4 19
           | -----------------
              4 4 19-17 

1 не является корнем, поэтому вы проверяете 2:

        2 | 4 0 15 -36
           | 8 16 62
           | -----------------
              4 8 31 26 

Увы, 2 тоже не рут.Но обратите внимание, что f (1) = −17 и f (2) = 26. У них противоположные знаков, что означает, что график пересекает ось x между x = 1 и x = 2, а корень находится между 1 и 2. (В данном случае это единственный root, поскольку вы определили, что существует один положительный корень и нет отрицательных корней.)

Единственный возможный рациональный корень между 1 и 2 — 3/2, и следовательно, либо 3/2 является корнем, либо корень иррационален. Вы пытаетесь 3/2 по синтетическому разделению:

        3/2 | 4 0 15 -36
             | 6 9 36
             | -----------------
                4 6 24 0 

Ура! 3/2 — это корень.Приведенный полином равен 4 x + 6 x + 24. Другими словами,

(4 x + 15 x — 36) ( х −3/2) = 4 x + 6 x + 24

Приведенный многочлен имеет степень 2, так что нет необходимости в большем методом проб и ошибок, и вы переходите к шагу 5.

Шаг 5. Теперь вы должны решить

4 x + 6 x + 24 = 0

Сначала разделите общий множитель 2:

2 x + 3 x + 12 = 0

Нет смысла пытаться множить этот квадратичный коэффициент, потому что вы определили, используя Правило знаков Декарта, что больше нет настоящие корни.Итак, вы используете квадратичный формула:

x = [−3 √9 — 4 (2) (12)] / 2 (2)

x = [−3 √ − 87] / 4

x = −3/4 (√87 / 4) i

Шаг 6. Помните, что вы нашли корень в более ранний шаг! Полный список корней —

3/2, −3/4 + (√87 / 4) я, −3/4 — (√87 / 4) я


Что нового

  • 19 октября 2020 г. : преобразовано в HTML5. Переменные, выделенные курсивом и имена функций; выделил мнимое i.
  • 3 ноября 2018 г. : Некоторые изменения форматирования для ясности, особенно с радикалами. Здесь отметили, что 0 является тройным корнем в этом примере.
  • (промежуточные изменения подавлены)
  • 15 февраля 2002 г. : Первая публикация.

Алгебра — нули / корни многочленов

Показать уведомление для мобильных устройств Показать все заметки Скрыть все заметки

Похоже, вы используете устройство с «узкой» шириной экрана ( i.е. вы, вероятно, пользуетесь мобильным телефоном). Из-за особенностей математики на этом сайте лучше всего просматривать в ландшафтном режиме. Если ваше устройство не находится в альбомном режиме, многие уравнения будут отображаться сбоку от вашего устройства (вы сможете прокручивать их, чтобы увидеть их), а некоторые пункты меню будут обрезаны из-за узкой ширины экрана.

Раздел 5-2: Нули / корни многочленов

Мы начнем этот раздел с определения, что такое корень или ноль многочлена.Мы говорим, что \ (x = r \) является корнем или нулем многочлена \ (P \ left (x \ right) \), если \ (P \ left (r \ right) = 0 \). Другими словами, \ (x = r \) является корнем или нулем многочлена, если он является решением уравнения \ (P \ left (x \ right) = 0 \).

В следующих двух разделах нам нужно будет найти все нули для данного многочлена. Итак, прежде чем мы перейдем к этому, нам нужно получить некоторые идеи относительно нулей многочленов, которые помогут нам в этом процессе.

Процесс нахождения нулей \ (P \ left (x \ right) \) на самом деле сводится к решению уравнения \ (P \ left (x \ right) = 0 \), и мы уже знаем, как это сделать. что для полиномов второй степени (квадратичных).2} = 0 \ hspace {0,25 дюйма} \ Rightarrow \ hspace {0,25 дюйма} \, \, \, \, \, \, \, \, \, x = 7 \]

Итак, этот многочлен второй степени имеет единственный ноль или корень. Также вспомните, что когда мы впервые посмотрели на них, мы назвали такой корень двойным корнем .

Мы решили каждую из них, сначала разложив многочлен на множители, а затем применив свойство нулевого множителя в факторизованной форме. Когда мы впервые посмотрели на свойство нулевого фактора, мы увидели, что оно гласит, что если произведение двух членов равняется нулю, то один из членов должен быть равен нулю для начала.

Свойство нулевого фактора может быть расширено до любого количества членов. Другими словами, если у нас есть произведение \ (n \) членов, равное нулю, то по крайней мере одно из них должно быть нулевым для начала. Итак, если бы мы могли разложить на множители многочлены более высокой степени, мы могли бы решить и их.

Давайте взглянем на пару из них. 2} \ left ({x + 5} \ right) \ left ({x + 4} \ right) \) Показать решение

С этим полиномом у нас есть четыре члена и нули здесь,

\ [x = — 5, \, \, \, x = — 1, \, \, \, x = 1, \, \, \, {\ mbox {and}} \, \, \, x = — 4 \]

Теперь у нас есть некоторая терминология, чтобы не мешать.2} \ left ({x + 5} \ right) \ left ({x + 4} \ right) \) Показать решение

Мы уже определили нули каждого из них в предыдущей работе или примерах в этом разделе, поэтому не будем переделывать эту работу. В каждом случае мы просто запишем ранее найденные нули, а затем вернемся к факторизованной форме многочлена, посмотрим на показатель степени каждого члена и дадим кратность.

a В этом случае у нас есть два простых нуля: \ (x = — 5, \, \, x = 3 \).

b Здесь \ (x = 7 \) — нуль кратности 2.

c У этого многочлена два нуля: \ (x = — 1 \) с кратностью 2 и \ (x = 2 \) с кратностью 3.

d В данном случае у нас три нуля. : \ (x = — 5 \) простой, \ (x = 0 \) с кратностью 4 и \ (x = 3 \) с кратностью 3.

e В последнем случае у нас четыре нуля. \ (x = — 5 \), который является простым, \ (x = — 1 \) с кратностью 3, \ (x = 1 \) с кратностью 2 и \ (x = — 4 \), который является простым.

Этот пример приводит нас к нескольким интересным фактам о многочленах. Вот первое и, наверное, самое главное.

Основная теорема алгебры

Если \ (P \ left (x \ right) \) — многочлен степени n , то \ (P \ left (x \ right) \) будет иметь ровно \ (n \) нулей, некоторые из которых могут повторяться .

Этот факт говорит о том, что если вы перечислите все нули и перечислите каждый \ (k \) раз, где \ (k \) — его кратность, у вас будет ровно \ (n \) чисел в списке.Другой способ сказать этот факт состоит в том, что кратность всех нулей должна складываться со степенью многочлена.

Мы можем вернуться к предыдущему примеру и проверить, что это верно для полиномов, перечисленных там.

Это будет приятный факт в нескольких разделах, когда мы подробно рассмотрим нахождение всех нулей многочлена. Если мы знаем верхнюю границу количества нулей для многочлена, тогда мы будем знать, когда мы все их найдем, и поэтому можем перестать искать.

Также обратите внимание, что некоторые нули могут быть сложными. В этом разделе мы работали с многочленами, у которых есть только действительные нули, но это не позволяет вам прийти к выводу, что эта теорема применима только к действительным нулям. Вполне возможно, что комплексные нули появятся в списке нулей.

Иногда бывает очень полезен и следующий факт.

Факторная теорема

Для многочлена \ (P \ left (x \ right) \),

  1. Если \ (r \) является нулем \ (P \ left (x \ right) \), то \ (x — r \) будет множителем \ (P \ left (x \ right) \).
  2. Если \ (x — r \) является множителем \ (P \ left (x \ right) \), то \ (r \) будет нулем \ (P \ left (x \ right) \).

Опять же, если мы вернемся к предыдущему примеру, мы увидим, что это подтверждается полиномами, перечисленными там.

Фактор-теорема приводит к следующему факту.

Факт 1

Если \ (P \ left (x \ right) \) является многочленом степени \ (n \) и \ (r \) является нулем \ (P \ left (x \ right) \), то \ (P \ left (x \ right) \) можно записать в следующем виде.

\ [P \ left (x \ right) = \ left ({x — r} \ right) Q \ left (x \ right) \]

, где \ (Q \ left (x \ right) \) — многочлен степени \ (n — 1 \). \ (Q \ left (x \ right) \) можно найти, разделив \ (P \ left (x \ right) \) на \ (x — r \).

Есть еще один факт, который нам нужно убрать с дороги.

Факт 2

Если \ (P \ left (x \ right) = \ left ({x — r} \ right) Q \ left (x \ right) \) и \ (x = t \) является нулем \ (Q \ left (x \ right) \), то \ (x = t \) также будет нулем \ (P \ left (x \ right) \).

Этот факт достаточно легко проверить напрямую. Во-первых, если \ (x = t \) является нулем \ (Q \ left (x \ right) \), то мы знаем, что

\ [Q \ left (t \ right) = 0 \]

, так как это значит быть нулем. Итак, если \ (x = t \) должен быть нулем \ (P \ left (x \ right) \), то все, что нам нужно сделать, это показать, что \ (P \ left (t \ right) = 0 \ ) и это на самом деле довольно просто. Вот она,

\ [P \ left (t \ right) = \ left ({t — r} \ right) Q \ left (t \ right) = \ left ({t — r} \ right) \ left (0 \ right) = 0 \]

, и поэтому \ (x = t \) является нулем \ (P \ left (x \ right) \).2} — 5x — 6 \) Факт 1 говорит нам, что мы можем записать \ (P \ left (x \ right) \) как,

\ [P \ left (x \ right) = \ left ({x — 2} \ right) Q \ left (x \ right) \]

и \ (Q \ left (x \ right) \) будет квадратичным многочленом. Затем мы можем найти нули \ (Q \ left (x \ right) \) любым из методов, которые мы рассмотрели до этого момента, и по факту 2 мы знаем, что два нуля, которые мы получаем из \ (Q \ left (x \ right) \) будет также нулями \ (P \ left (x \ right) \). На этом этапе у нас будет 3 нуля, и на этом все готово.

Итак, давайте найдем \ (Q \ left (x \ right) \). Для этого все, что нам нужно сделать, — это быстрое синтетическое разделение, как показано ниже.

\ [\ begin {align *} \ left. {\ underline {\, 2 \,}} \! \ право | & \, \, \, \ begin {array} {* {20} {l}} 1 & {\, \, 2} & {- 5} & {- 6} \ end {array} \\ & \, \ , \, \, \, \, \ underline {\, \, \ begin {array} {* {20} {l}} {} & 2 & {\, \, \, \, 8} & {\, \, \, \, \, 6} \ end {array}} \\ & \, \, \, \, \, \ begin {array} {* {20} {l}} {\, 1} & {4} & {\, \, 3} & {\, \, \, \, \, 0} \ end {array} \ end {align *} \]

Перед записью \ (Q \ left (x \ right) \) вспомните, что последнее число в третьей строке — это остаток, и что мы знаем, что \ (P \ left (2 \ right) \) должно быть равно этому номер.Итак, в этом случае мы имеем \ (P \ left (2 \ right) = 0 \). Если задуматься, мы уже должны знать, что это правда. В постановке задачи нам указывалось, что \ (x = 2 \) является нулем \ (P \ left (x \ right) \), а это означает, что мы должны иметь \ (P \ left (2 \ right) = 0 \).

Итак, зачем об этом говорить? Это отличная проверка нашего синтетического подразделения. Поскольку мы знаем, что \ (x = 2 \) является нулем для \ (P \ left (x \ right) \), и мы получаем любое другое число, кроме нуля в этой последней записи, мы будем знать, что мы сделали что-то не так, и мы можем вернуться и найти ошибку.2} — 5х — 6 \). Подставляя факторизованную форму \ (Q \ left (x \ right) \) в \ (P \ left (x \ right) \), получаем

\ [P \ left (x \ right) = \ left ({x — 2} \ right) \ left ({x + 3} \ right) \ left ({x + 1} \ right) \]

Кстати, так факторизовались многочлены в первом наборе примеров. Для этого потребуется немного больше работы, но это можно сделать таким же образом.

Урок 39: Нули полиномиальных функций, часть II:
Верхняя и нижняя границы, теорема о промежуточном значении, основная теорема алгебры и теорема о линейной факторизации

Цели обучения


После изучения этого руководства вы сможете:
  1. Определите, является ли данное число верхней или нижней границей корней многочлен функция.
  2. Используйте теорему о промежуточном значении, чтобы аппроксимировать действительные нули многочлен функции.
  3. Знайте, что если не действительное комплексное число является корнем многочлена функция что его сопряжение также является корнем.
  4. Знать, что такое основная теорема алгебры.
  5. Используйте теорему о линейной факторизации, чтобы найти многочлен n-й степени функция учитывая его нули.

Введение



В этом уроке мы рассмотрим несколько аспектов иметь дело с нули полиномиальных функций. Если вам нужен обзор того, как найти нули, теорема рационального нуля или правило знаков Декарта, бесплатно перейти к Урок 38: Нули Полиномиальный Функции, часть I. На этой странице мы немного углубимся в понятие нулей.Одна вещь, которую мы рассмотрим, — это найти в оценки сверху и снизу корней полиномиальной функции. Этот может помочь нам сузить возможности рациональных нулей. Другой Концепция на этой странице — это теорема о промежуточном значении. Это может помочь сузить возможности реальных нулей, особенно тех, которые земля между целыми значениями. Так же будем работать с ненастоящими сложный числа. Знаете ли вы, что если не действительное комплексное число равно нулю полиномиальной функции, что и ее сопряженная? Мы будут следовать за этим, используя фундаментальную теорему алгебры и Линейный Теорема факторизации, чтобы найти полиномиальные функции с учетом нулей. Ух ты, Похоже, у нас уже есть работа. Я думаю тебе лучше начать.

Учебник




Верхняя и нижняя границы Теорема .

Верхняя граница
Если разделить полиномиальную функцию f ( x ) по ( x c ), где c > 0, используя синтетическое деление, и это дает все положительные числа, тогда c — это верхняя граница действительных корней уравнения f ( x ) = 0.

Обратите внимание, что для c to быть верхней границей. Один — c > 0 или положительный. Во-вторых, все коэффициенты частного, а также остаток положительные.


Нижняя граница
Если разделить полиномиальную функцию f ( x ) по ( x c ), где c <0, используя синтетическое деление, и это дает чередующиеся знаки, тогда c является нижней границей действительных корней уравнения f ( x ) = 0.Особое внимание следует обратить на то, что нули могут быть либо положительными, либо отрицательный.

Обратите внимание, что для c to быть нижней границей. Один — c <0 или же отрицательный. Другой заключается в том, что последовательные коэффициенты частное а остальные имеют чередующиеся знаки.




Пример 1 : Покажите, что все действительные корни уравнения лежат между — 4 и 4.

Другими словами, нам нужно показать, что — 4 — это меньшее связаны и 4 оценка сверху действительных корней данного уравнения.

Проверка Нижняя граница:
Давайте применим синтетическое разделение с — 4 и посмотрим, получим ли мы чередующиеся знаки:

Обратите внимание, как c = -4 <0 И последовательные знаки в нижнем ряду нашего синтетического дивизиона заместителя .

Вы знаете, что это значит?

— 4 — нижняя граница действительных корней этого уравнение.


Проверка Верхняя граница:
Давайте применим синтетическое деление с 4 и посмотрим, получим ли мы все положительные результаты:

Обратите внимание, как c = 4> 0 И все знаков в нижнем ряду нашего синтетического подразделения положительный.

Вы знаете, что это значит?

4 — верхняя граница действительных корней этого уравнение.

Поскольку — 4 — нижняя граница, а 4 — верхняя граница для настоящие корни уравнения, то это означает, что все действительные корни уравнения лежат от — 4 до 4.


Промежуточная ценность Теорема .

Если f ( x ) — это полиномиальная функция и f ( a ) и f ( b ) имеют разные знаки, то есть не менее один значение, c ,
между a и b таким образом, что f ( c ) = 0.


Другими словами, когда у вас есть полиномиальная функция и одно входное значение заставляет функцию быть положительной, а другую — отрицательной, тогда там имеет быть по крайней мере одним значением между ними, которое вызывает многочлен функция быть 0.

Это работает, потому что 0 отделяет плюсы от негативы. Итак, чтобы перейти от положительного к отрицательному или наоборот, вам придется ударить промежуточная точка, проходящая через 0.




Пример 2 : Показать, в котором реальный ноль находится в диапазоне от 2 до 3. Используйте промежуточное значение теорема чтобы найти приближение этого нуля с точностью до десятых.

При поиске функциональных значений вы можете использовать синтетическое подразделение или напрямую вставьте номер в функцию.Поскольку мы будем только интересно узнать функциональную ценность этой проблемы, я идущий чтобы напрямую подключить мое значение x к функция. Если бы мне нужно было больше, например, знаки частного, например выше, тогда я бы использовал синтетическое деление.

Находка ф (2):

Находка ф (3):

Так как есть изменение знака между f (2) = -2 и f (3) = 5, то согласно Intermediate Теорема о значении , существует по крайней мере одно значение от 2 до 3, которое является нулем этой полиномиальной функции.

Проверка функциональных значений с интервалом в одну десятую для смены знака:


Находка ф (2.1):

Находка ф (2.2):

Находка ф (2.3):

Находка ф (2.4):

Находка ф (2,5):

Привет, у нас поменяна вывеска !!!!!

Теперь мы хотим найти ноль с точностью до десятых. Так оно и есть должно быть x = 2,4 или x = 2,5. Мы не всегда можем ориентироваться на функциональную ценность ближе до нуля.

Нам нужно будет копнуть немного глубже и пройти мимо интервалы сотых:



Уф !!!! Наконец мы подошли к смене знака между последовательный сотые.Это означает, что мы немного сузили его лучше. Между 2,44 и 2,45 находится ноль.

Поскольку он приземлится чуть ниже 2,45, ближайший десятый будет быть 2,4.

Работа здесь несложная, просто утомительная.




Основная теорема Алгебра

Каждое уравнение полиномиальной функции f ( x ) = 0 степени один или выше имеет по крайней мере один комплексный корень.


Имейте в виду, что комплексные числа включают действительные числа. Настоящий числа — это комплексные числа, мнимая часть которых равна 0.



В уравнениях полиномиальных функций невещественные комплексные корни всегда происходит в сопряженных парах.

Другими словами, если комплексное число с мнимой часть является корнем уравнения полиномиальной функции, то сопряженная к нему также является корнем та же функция.

Помните, что сопряжение a + bi есть a bi . Итак, если 2 + 3 i — известный корень многочлен уравнение функции, то 2 — 3 i тоже.

Если вам нужен обзор комплексных чисел, не стесняйтесь идти to Учебное пособие 12: Комплексные числа.


Линейная факторизация Теорема .

Если

где n > 1 и

, затем

где комплексные числа
(возможно, действительные и не обязательно отчетливый)


Другими словами, полиномиальная функция степени n , где n > 0, можно разложить на множители на n (не обязательно различных) линейных множителей над комплексным числом поле.

Имейте в виду, что некоторые факторы могут иметь место более чем в одном время. Для каждый раз, когда появляется линейный коэффициент, он считается линейным фактор. Например, если, линейный коэффициент ( x + 2) имеет множественность из 3, что означает, что множитель встречается трижды. Так технически есть 4 линейных фактора, один ( x — 3) а также три ( x + 2).Это соответствует степень полиномиальной функции.




Пример 3 : Используйте данный корень, чтобы найти все корни многочлен уравнение; 1 + и .

Поскольку комплексное число 1 + i является корнем, это означает, что конъюгат 1 — i также является корнем.Это поможет нам разбить функцию, чтобы найти любые другие корни.

Это делается так же, как когда вам дают настоящий ноль.

Если вам нужен обзор по поиску корней многочлена уравнение f ( x ) = 0 при получении root-прав можно переходить к Tutorial 37: Синтетическое деление и теоремы об остатке и множителях.

Используя синтетическое деление, найдите частное, которое мы получить:

Фу !!! Посмотрите на все эти комплексные числа в частном. Не бойтесь, когда мы добавляем в наш конъюгат, используя это частное, те сложный числа исчезнут, и у нас останется хорошее частное с настоящий числовые коэффициенты.

Проверьте это:


Теперь мы куда-то идем.Отсюда мы можем переписать оригинал проблема с использованием корней, которые у нас есть выше, и частного, который мы закончился в этом последнем синтетическом дивизионе.



* Первые два множителя равны x минус комплексные нули
* Третий множитель — это частное найдено напрямую выше

* Данный комплексный ноль

* Сопряженный ноль

* Установка 3-го фактора = 0


Итак, корни полиномиального уравнения равны 1 + i , 1 — и и -3/5.




Пример 4 : Фактор а) как произведение факторов, неприводимых над рациональными числа б) как произведение множителей, которые не сводятся к действительным числам, а также в) в полностью факторизованной форме, включающей комплексные не действительные числа.

Фактор как произведение факторов, которые неприводимый над рациональным номера:



Так как 11 не является идеальным квадратом, это, насколько мы может это учитывать используя только рациональные числа.

Фактор как произведение несводимых факторов над вещественными числами:



Знаете ли вы, что сумму квадратов можно разложить на множители? над комплексом нереальные числа как?

Полностью факторизованная форма, включающая сложные нереальные номера:


Обратите внимание на то, как однажды мы провели факторизацию комплексных чисел что мы закончили с четырьмя линейными множителями и что наш многочлен был четвертой степени.




Создание полинома Функция при наличии Нули

Теперь обратимся. В следующий два примера, нам будут даны нули и степень полиномиальной функции, и мы нужно будет узнать, что это за многочлен.

Шаг 1. Используйте данный нулей и теоремы о линейной факторизации, чтобы выписать все факторы полиномиальной функции.

Имейте в виду, что если вам дадут ненастоящий комплексный ноль, что его сопряженный также является нулем.

Также имейте в виду, что степень говорит вам, как много линейных факторов над комплексными числами (возможно, действительными и не обязательно различными) что у вас будет.

Коэффициенты записываются следующим образом: если c — ноль, то ( x c ) является фактором полиномиальной функции.

Шаг 2: Умножьте все факторы, найденные на шаге 1.




Пример 5 : Найдите n -ю степень многочлен функция, где n = 3; 2 + 3 i и 4 нули; f (3) = -20.


Поскольку наша степень 3, это означает, что есть три линейные коэффициенты над комплексные числа (возможно, действительные и не обязательно различные).

Обратите внимание, что нам даны только два нуля. Нам нужно придумать третий. У тебя есть идеи?

Ах да, если не действительное комплексное число равно нулю, то его конъюгат тоже ноль.Поскольку 2 + 3 i является нуль, это означает, что 2 — 3 i также равно нулю.

Используя теорему о линейной факторизации , мы получить:


Шаг 2: Умножить все факторов, найденных на шаге 1.



* Расст.- через комп. числа

* Умножить комп. факторы
* Упростить ( i в квадрате = -1)
* Умножение оставшихся множителей


Эта проблема привела к другому условию: f (3) = -20.

Это поможет нам найти в этой проблеме.


* f (3) = -20

* Решить для a sub n


Собирая все вместе, получаем:


Практические задачи



Это практические задачи, которые помогут вам перейти на следующий уровень. Это позволит вам проверить и понять, понимаете ли вы эти типы проблем. Математика работает как и все в противном случае, если вы хотите добиться успеха в этом, вам нужно практиковать это. Даже лучшие спортсмены и музыканты получали помощь и много практиковаться, практиковаться, практиковаться, чтобы хорошо освоить свой вид спорта или инструмент На самом деле не бывает слишком много практики.

Чтобы получить от них максимальную отдачу, вы должны решить проблему свой, а затем проверьте свой ответ, щелкнув ссылку для ответа / обсуждения для этой проблемы .По ссылке вы найдете ответ а также любые шаги, которые позволили найти этот ответ.

Практика Задача 1а: Показать, что все настоящие корни данное уравнение лежат между -3 и 4.

Практика Задача 2a: Покажите, что данный многочлен имеет настоящий ноль между заданные целые числа.Используйте теорему о промежуточном значении, чтобы найти аппроксимация этого нуля с точностью до десятых.

Практика Проблема 3a: Используйте заданный корень, чтобы найти все корни заданное полиномиальное уравнение.

Практика Задача 4a: Разложите данный многочлен на множители функция а) как произведение множителей, неприводимых по рациональным числам, б) при в произведение множителей, которые не сводятся к действительным числам, и c) в полностью факторизованная форма, включающая комплексные не действительные числа.

Практика Задача 5a: Найдите многочлен n -й степени функционировать с в данных условиях

Нужна дополнительная помощь по этим темам?






Последний раз редактировал Ким Сьюард 15 марта 2012 г.
Авторские права на все содержание (C) 2002 — 2012, WTAMU и Kim Seward. Все права защищены.

Определите степень и старший коэффициент полиномиальных функций

Из-за формы полиномиальной функции мы можем видеть бесконечное разнообразие в количестве членов и степени переменной. Хотя порядок членов в полиномиальной функции не важен для выполнения операций, мы обычно располагаем члены в порядке убывания степени или в общем виде. градусов полинома — это наивысшая степень переменной, которая встречается в полиноме; это мощность первой переменной, если функция имеет общий вид. Главный член — это член, содержащий наивысшую степень переменной или член с наивысшей степенью. Старший коэффициент — это старший член.

Общее примечание: Терминология полиномиальных функций

Рисунок 6

Мы часто переставляем многочлены так, чтобы их степени были убывающими.

Когда многочлен записывается таким образом, мы говорим, что он имеет общий вид.

Практическое руководство. Для заданной полиномиальной функции определите степень и старший коэффициент.

  1. Найдите наибольшую степень x
    , чтобы определить градусную функцию.
  2. Определите член, содержащий наибольшую степень x
    , чтобы найти главный член.
  3. Определите коэффициент главного члена.

Пример 5: Определение степени и ведущего коэффициента полиномиальной функции

Укажите степень, старший член и старший коэффициент следующих полиномиальных функций.{6} + 2x — 6 \\ [/ латекс].

Решение

Определение конечного поведения полиномиальных функций

Знание степени полиномиальной функции помогает нам предсказать ее конечное поведение. Чтобы определить его конечное поведение, посмотрите на главный член полиномиальной функции. Поскольку степень главного члена самая высокая, этот член будет расти значительно быстрее, чем другие члены, поскольку x становится очень большим или очень маленьким, поэтому его поведение будет доминировать на графике.{3} \\ [/ латекс]

Пример 6: Определение конечного поведения и степени полиномиальной функции

Опишите поведение конца и определите возможную степень полиномиальной функции на рисунке 7.

Рисунок 7

Решение

Поскольку входные значения x становятся очень большими, выходные значения [latex] f \ left (x \ right) \\ [/ latex] неограниченно увеличиваются. Поскольку входные значения x становятся очень маленькими, выходные значения [latex] f \ left (x \ right) \\ [/ latex] уменьшаются без ограничений.Мы можем символически описать конечное поведение, написав

[латекс] \ begin {case} \ text {as} x \ to — \ infty, f \ left (x \ right) \ to — \ infty \\ \ text {as} x \ to \ infty, f \ left (x \ right) \ to \ infty \ end {case} \\ [/ latex]

На словах можно сказать, что, когда значения x приближаются к бесконечности, значения функций приближаются к бесконечности, а когда значения x приближаются к отрицательной бесконечности, значения функций приближаются к отрицательной бесконечности.

Мы можем сказать, что этот график имеет форму степенной функции нечетной степени, которая не была отражена, поэтому степень полинома, создающего этот график, должна быть нечетной, а старший коэффициент должен быть положительным.{4} \\ [/ латекс]; следовательно, степень полинома равна 4. Степень четная (4), а старший коэффициент отрицательный (–3), поэтому конечное поведение равно

.

[латекс] \ begin {case} \ text {as} x \ to — \ infty, f \ left (x \ right) \ to — \ infty \\ \ text {as} x \ to \ infty, f \ left (x \ right) \ to — \ infty \ end {case} \\ [/ latex]

Попробовать 5

Дана функция [латекс] f \ left (x \ right) = 0,2 \ left (x — 2 \ right) \ left (x + 1 \ right) \ left (x — 5 \ right) \\ [/ latex] , выразить функцию как полином в общем виде и определить главный член, степень и конечное поведение функции.

Решение

Определение локального поведения полиномиальных функций

Помимо конечного поведения полиномиальных функций, нас также интересует, что происходит в «середине» функции. В частности, нас интересуют места, где меняется поведение графа. Поворотная точка — это точка, в которой значения функции изменяются от увеличения к уменьшению или от уменьшения к увеличению.

Фиг.10

Нас также интересуют перехваты.Как и во всех функциях, точка пересечения y- — это точка, в которой график пересекает вертикальную ось. Точка соответствует паре координат, в которой входное значение равно нулю. Поскольку многочлен является функцией, каждому входному значению соответствует только одно выходное значение, поэтому может быть только один перехватчик y- [latex] \ left (0, {a} _ {0} \ right) \\ [/ latex ]. Перехваты x- происходят при входных значениях, которые соответствуют нулевому выходному значению. Возможно иметь более одного перехвата x-.

Общее примечание: точки пересечения и поворотные точки полиномиальных функций

Поворотная точка графика — это точка, в которой график меняет направление с увеличения на уменьшение или с уменьшения на увеличение. Перехват y- — это точка, в которой функция имеет нулевое входное значение. x -перехват — это точки, в которых выходное значение равно нулю.

Как: для заданной полиномиальной функции определить точки пересечения.

  1. Определите точку пересечения y- , установив [latex] x = 0 \\ [/ latex] и найдя соответствующее выходное значение.
  2. Определите перехват x , вычислив входные значения, которые дают нулевое выходное значение.

Пример 8: Определение точек пересечения полиномиальной функции

Дана полиномиальная функция [латекс] f \ left (x \ right) = \ left (x — 2 \ right) \ left (x + 1 \ right) \ left (x — 4 \ right) \\ [/ latex] , записанный в факторизованной форме для вашего удобства, определите интервалы перехвата y и x .

Решение

Перехват y- происходит, когда входной сигнал равен нулю, поэтому замените 0 на x .

[латекс] \ begin {case} f \ left (0 \ right) = \ left (0-2 \ right) \ left (0 + 1 \ right) \ left (0-4 \ right) \ hfill \\ \ текст {} = \ left (-2 \ right) \ left (1 \ right) \ left (-4 \ right) \ hfill \\ \ text {} = 8 \ hfill \ end {case} \\ [/ latex]

Перехватчик y- равен (0, 8).

-Перехват x происходит, когда выходной сигнал равен нулю.

[латекс] \ begin {case} \ text {} 0 = \ left (x — 2 \ right) \ left (x + 1 \ right) \ left (x — 4 \ right) \ hfill \\ x — 2 = 0 \ hfill & \ hfill & \ text {или} \ hfill & \ hfill & x + 1 = 0 \ hfill & \ hfill & \ text {или} \ hfill & \ hfill & x — 4 = 0 \ hfill \\ \ text {} x = 2 \ hfill & \ hfill & \ text {или} \ hfill & \ hfill & \ text {} x = -1 \ hfill & \ hfill & \ text {или} \ hfill & \ hfill & x = 4 \ end {case} [/ latex]

x -перехват: [latex] \ left (2,0 \ right), \ left (-1,0 \ right) \\ [/ latex] и [latex] \ left (4,0 \ right) )\\[/латекс]. {2} -20x \\ [/ latex], определите y — и x — перехватывает.

Решение

Сравнение гладких и непрерывных графиков

Степень полиномиальной функции помогает нам определить количество точек пересечения x и количество точек поворота. Полиномиальная функция n -й степени является произведением n факторов, поэтому она будет иметь не более n корней или нулей, или x -перехваченных значений. График полиномиальной функции степени n должен иметь не более n — 1 точек поворота.Это означает, что у графика самое большее на одну точку поворота меньше, чем степень многочлена, или на одну меньше, чем количество факторов.

Непрерывная функция не имеет разрывов в графике: график можно нарисовать, не отрывая перо от бумаги. Плавная кривая — это график без острых углов. Поворотные точки гладкого графика всегда должны приходиться на закругленные кривые. Графики полиномиальных функций бывают как непрерывными, так и гладкими.

Общее примечание: точки пересечения и поворотные точки многочленов

Полином степени n будет иметь не более n x -перехватов и n — 1 точек поворота.{3} \\ [/ латекс]

Решение

Пример 11: Выводы о полиномиальной функции из графика

Какой мы можем сделать вывод о полиноме, представленном графиком, показанным на графике на рисунке 13, на основе его пересечений и точек поворота?

Рисунок 13

Решение

Рисунок 14

Конечное поведение графа говорит нам, что это граф полинома четной степени.

График имеет 2 точки пересечения x , что соответствует степени 2 или больше, и 3 точки поворота, что указывает на степень 4 или больше.Исходя из этого, было бы разумно сделать вывод, что степень ровная и не менее 4.

Попробуй 8

Что мы можем сделать о полиноме, представленном на рисунке 15, на основании его точек пересечения и точек поворота?

Рисунок 15

Решение

Пример 12: Выводы о полиномиальной функции по факторам

Учитывая функцию [латекс] f \ left (x \ right) = — 4x \ left (x + 3 \ right) \ left (x — 4 \ right) \\ [/ latex], определите локальное поведение.

Решение

Перехват y находится путем вычисления [latex] f \ left (0 \ right) \\ [/ latex].

[латекс] \ begin {case} f \ left (0 \ right) = — 4 \ left (0 \ right) \ left (0 + 3 \ right) \ left (0-4 \ right) \ hfill \ hfill \ \ \ text {} = 0 \ hfill \ end {case} \\ [/ latex]

Перехват y — [латекс] \ left (0,0 \ right) \\ [/ latex].

Перехваты x находятся путем определения нулей функции.

[латекс] \ begin {case} 0 = -4x \ left (x + 3 \ right) \ left (x — 4 \ right) \\ x = 0 \ hfill & \ hfill & \ text {или} \ hfill & \ hfill & x + 3 = 0 \ hfill & \ hfill & \ text {или} \ hfill & \ hfill & x — 4 = 0 \ hfill \\ x = 0 \ hfill & \ hfill & \ text {или} \ hfill & \ hfill & \ text {} x = -3 \ hfill & \ hfill & \ text {или} \ hfill & \ hfill & \ text {} x = 4 \ end {case} \\ [/ latex]

x -перехват: [latex] \ left (0,0 \ right), \ left (-3,0 \ right) \\ [/ latex] и [latex] \ left (4,0 \ right) )\\[/латекс].

Степень равна 3, поэтому график имеет не более 2 поворотных точек.

Попробуй 9

Дана функция [латекс] f \ left (x \ right) = 0,2 \ left (x — 2 \ right) \ left (x + 1 \ right) \ left (x — 5 \ right) \\ [/ latex] , определить местное поведение.

Решение

1. Каково наименьшее и наибольшее количество различных действительных корней многочлена 6-й степени?

1. У многочлена не может быть больше корней, чем степень. Итак, полином шестой степени имеет не более 6 различных действительных корней.Например, (x-1) (x-2) (x-3) (x-4) (x-5) (x-6) имеет степень 6 и 6 различных действительных корней.

У многочлена не может быть действительных корней. Таким образом, наименьшее количество действительных корней многочлена степени 6 может быть 0. Это было бы так, если бы график y = polynomial не имел пересечений по x. Например, x 6 +1 имеет степень 6 и не имеет реальных корней [а график

f (x) = x 6 +1 не имеет точек пересечения по оси x].

————————————————- ————————————

2.Чтобы найти i в степени положительного целого числа, разделите степень на 4 и используйте остаток как степень. Итак, если мы разделим 103 на 4, остаток будет 3. Следовательно, i 103 = i 3 = -i.

————————————————- ———————————-

3. x 2 -8x + 17 = 0 x = [8 ± √ (-4)] / 2 = [8 ± 2i] / 2 = 4 ± i

————————————————- ———————————-

4.Согласно теореме об остатке, если многочлен f (x) с действительными коэффициентами делится на x — c, то остаток равен f (c).

Итак, если f (x) = x 8 — 1, то остаток от деления f (x) на x-2 равен f (2) = 2 8 — 1 = 255.

————————————————- ———————————

5. Если f (x) имеет рациональный корень, p / q, то p является делителем постоянного члена, а q является делителем главного коэффициента.

Добавить комментарий

Ваш адрес email не будет опубликован. Обязательные поля помечены *